Você está na página 1de 103

MCCQE

Medical Council of Canada Qualification Examination






1
1. A 12 lead ECG showing very mild delta waves with a short P interval. !ther
wise the ECG has no a"normality. #hat is the diagnosis$
A% #olf Par&inson #hite syndrome
"% '()
c% left axis deviation
d% right axis deviation with "undle "ranch "loc&
e%
2. A 2 lead ECG showing a fast rhythm P%wave *resents and the rhythm is regular.
+his is an ECG of a lady who is com*laining of *al*itation. #hat is the diagnosis$
a% sinus tachycardia
"% atrial tachycardia
c% atrial flutter
d% nodal tachychardia with left axis deviation
e% ,(+
-. Picture of a child with a swelling in the um"ilical area. #hat are you going to do$
a% reassess in 12 month time
"% urgent surgery
c% x%ray
.. Picture of a foot of a child with very well circumscri"ed erythematous lesions in
the sole of the foot. +he lesions are itchy and similar lesions are *resent in the
u**er extremities/ and the child is having fever. #hat is the diagnosis$
a% Coxsac&ie virus
"% ,arco*tes sca"iei
c% molluscum
d% stre*tococcus e*idermis
e% CM(
0. C1 of a new"orn showing a hy*odensity in the left side. +he child is in
res*iratory distress. #hat is your initial management$
a% needle as*iration of the left lung
"% intu"ation and ventilation
c% ex*ectant thera*y
d%
e%
2. C1 of a neonate few hours after "irth. +he x%ray showing a full a"dominal and
chest view 3very "ad 4uality x%ray5. +he a"normality in the chest is of a large
dense central shadow/ which may re*resent an infiltration in the lungs or a large
heart. +he child is -2%w& gestation and soon after "irth he develo*ed 6,. #hat is
your diagnosis$
a% hyaline mem"rane disease
2
"% congenital heart disease
c% *neumonia
d% *ersistent fetal circulation
7. A *icture of du*eturan8s contracture
9. A *icture of an infant who is s*reading his legs and showing his testicles/ which
loo&/ red and swollen. +he child is crying and on examination there is no "owel
sound over them and the child screams when they are elevated. #hat are you
going to do for him$
a% urgent ex*loration
"% "oo& him for surgery
c% reassure the mother and send home
d% :;, of testicles
e%
<. 1%ray of a &nee of a 2%yr. old child who has not wal&ed yet. =t shows the ti"ia/
which has a wide cu**ing at its *roximal end with o"scure meta*hesial *late.
+here is no significant family history of any disease. #hat is wrong with child$
a% ic&ets
"% achondro*lasia
c% hy*er *arathyroidism
d% hy*erclacemia

1>. A *icture of an arthro*athy mutilan of the left hand. +here is a nodule over the 2
nd
metacar*o*halangeal ?oint. +his man has an acute exacer"ation of his condition/
and he has similar *ro"lem in his "ig toe. #hat is your immediate management$
a% Allo*uranol
"% colchecine
c% indomithacine
d% acetamino*hen
e%
11. A *icture of a young female centered on her nec&/ showing a swelling in the
midline that moves id when she *rotrudes her tongue. #hat is the diagnosis$
a% thyroglossal cyst
"% "ranchial cyst
c% thyroid nodule
d%
12. A *icture of a new"orn "a"y girl/ showing an enlarged clit/ and la"iae/ and
"leeding *er vagina. @ou are the doctor in the ward and the charge nurse as&s you
to come and see this. #hat is your next ste*$
a% Call a gynecologist
"% as& the mother if she too& 6E, during the *regnancy
c% do an urgent :;, of lower a"domen
-
d% o"serve and wait
SURGERY
1-. A 20 years old lady was admitted for emergency hemicolectomy. ,he was *re*ared
for the o*eration and was given =( fluids and anti"iotics. 6uring the induction of
the anesthesia she develo*ed hy*otension. #hat is the reason of her hy*otension$
a%
volume loss
"% allergy to anti"iotic
c% cardiogenic shoc&
d%
1.. Aest test for initial diagnosis of colorectal carcinoma isB
i% P
10. A *t *resented with history of fatigue and wt loss/ *artial *tosis of the left eye/ area
of dry s&in over the chee& and small *u*il in the same side as well as a sun&en left
eye. #hat is the cause of this condition$
a% "roncogenic carcinoma
"% internal carotid artery insufficiency
c%cervical ri"
d% thoracic outlet syndrome
12. A female com*laining of right flan& *ain radiating to the groin with
microhematuria. @ou did an =(P and find a - mm stone in the lower ureter. #hat is
the "est management$
a% fluids and analgesia
"% *ercutaneous *yelostomy

17. A *t came with a head trauma/ after a lucid interval he develo*ed coma. #hat is
the cause of his coma$
a% Ex*anding extradural hematoma
"% "rain edema
c% su"dural hematoma
d% interventricular hemorrhage
e% "rain concussion
19. Pt diagnosed to have a volvulus segment/ what is the "est management$
a% de%rotation with sigmoidosco*y
"% Aa enema under *ressure
c% hartmanCs *rocedure
1<. A 2< year%old female *t with many years heart"urn/ es*ecially after meals/ the *ain
is relieved when setting/ and there is an associated regurgitation. #hat is the
diagnosis$
.
a% hiatus hernia
"% *e*tic ulcer disease
c% cholelithaiasis
d% *yloric stenosis
2>. All of the following can cause laryngeal *oly*s exce*tB
a% *resent medications
"% vocal *ursuit
21. A lady went for an o*eration and 2 hours after she came "ac& from the theatre she
develo*ed hy*otension. #hat is the cause of her *ro"lem
a% volume loss
"% allergy to a "lood *roduct
c% cardiogenic shoc&
22. #hich one of the following is the "est indicator for a cricothyroidotomy$
a% a massive face smash
"% C0%C2 fracture
c%
d%
2-. A *t sustained an in?ury to his hand with a rusty nail. #hat is the "est and most
efficient way to *revent gas gangrene$
a% clean the wound thoroughly
"% give tetanus toxoid and immunoglo"ulins
c% *ut it in a hy*er"aric oxygen
d%
2.. Pt *ost hemicolectomy develo*s *ain and tenderness over the medial as*ect of the
thigh/ which is tender to touch. #hat is the diagnosis$
a% su*erficial throm"o*hle"itis
"% dee* vain throm"osis
c% cellulites
d%

20. A .9%yr. old man/ who is a &nown *t with duodenal ulcer/ came with an acute
e*igastric *ain radiating through to the "ac&. #hat is the diagnosis$
a% *erforation
"% *enetration
c% a**endicitis
d%
22. #hich of the following thyroid cancers has the "est *rognosis$
a% Pa*illary carcinoma
"% follicular carcinoma
c% a*lastic carcinoma
0
d% congenital carcinoma
e% medullary carcinoma
27. Pt cannot elevate his arm "etween <>%12> degrees. #hat is wrong with him$
a% Aici*ital tendonitis
"% rotator cuff muscle tear
c% "ursitis

29. A *t with ->D first degree "urns. @ou will give him all of the following exce*tB
a% tetanus toxoid
"% =( normal saline
c% Pac&ed AC
d%
2<. An industrial wor&er had an in?ury to the *ul* of his index finger/ came one day
later with *ain from the ti* of his finger extending to the flexor area in his *alm
with an erythematous area around it/ and *ain on *assive extension of the fingers.
#hat is your management$
a% =( anti"iotics
"% incision and drainage and =( anti"iotics
c% tendon sheath ex*loration/ washout and =( anti"iotics

->. A 1-%yr. old girl came to see you for a *ro"lem in her "ac&. ,he has a 2> degrees
scoliosis of the lower "ac&/ a tuft of hair in the left "uttoc& area and wasting of the
left thigh muscles. #hat are you going to do for her$
a% re x%ray in one year
"% surgical correction
c% lower "ac& "rac&et
d% myelogra*hy
e%

-1. A 2> years old female *resented to you with *ainless hematuria and *assing of
clots/ with fre4uency/ nocturia. #hat is the diagnosis$
a% "ladder CA
"% cystitis
-2. A 7>%yr. old man com*laining of *ain in his mouth. @ou examine him and he has a
flat ulcer on the gum of the lower ?aw/ near the molar teeth area. )e tills you that
his denture has recently "ecome loose and ill fitting. #hat is the diagnosis$
a% traumatic ulcer
"% ulcerating carcinoma of the "uccal mucosa
c%
2
d%

--. )ow can you detect a fracture of the head of the radius in young "oy
a% as&ing him to do *ronation and su*ination of the arm
"%
c%
-.. Pt *ost hemicolectomy and reanastomosis of "owel/ develo*ed fever - days *ost
o*. #hat is the most li&ely cause$
a% wound infection
"% :+=
c% anastomosis lea&
d%
-0. Post +A a *t develo*ed "leeding from his external urethral meatus. )ow are
going to manage this *t$
a% =(P and su*ra*u"ic Eoley catheter
"% immediate su*ra*u"ic Eoley catheter.
-2. A *t with red eye/ irregular *u*il/ ocular *ain. #hat is the diagnosis$
a% acute iritis
"% acute glaucoma
c%

-7. =n uvietis you will find all of the following exce*t
a% discharge from the eye
"% infiltrate in the anterior cham"er
c% redness of the lim"us
d% small irregular *u*il
e%
-9. Pt was &e*t on the ventilator. )is PaC!2F0./ Pa!2F92/ )C!-F20. #hat are
going to change for him$
a% res*iratory rate
"% tidal volume
c%
d%
-<. A lady *ost*artum and "reast%feeding develo*ed swelling/ redness and heat over
the right calf muscle/ with mild fever. )ow are you going to manage her$
a% =( he*arin
"% oral warfarin
c% =( anti"iotic
d% early mo"iliGation
7
.>. 7> yo lady develo*ed nystagmus/ and no tennitus. ,he gets diGGy every time she
moves her head in any *osition. #hat is the diagnosis$
a% 'a"rinthitis
"% meniers disease
c% acoustic neuroma
d%
e%
.1. A 2< years old man develo*ed acute onset a"dominal *ain/ colic&y in nature and
around the um"ilical area/ associated with increased tym*anic "owel sounds. #hat
is most li&ely diagnosis$
a% cecal volvulus
"% intestinal o"struction
c%
d%
.2. A *t develo*ed a su*racondylar fracture of the right arm. Eollowing it he could not
extend his wrist. #hat is the damaged nerve$
a% adial nerve
"% ulnar nerve
c% median nerve
d% cutanueos anti"rachii nerve

.-. !ld lady with acute "owel o"struction and diagnosed with small "owel ischemia.
#hat do you ex*ect to find$ a%
raised amylase level
"% slight "lood P
c%
d%
... A female *t comes to the hos*ital with re*eated attac&s of hy*oglycemia. !n
investigation you find high levels of serum insulin and low levels of serum C%
*e*tide. @our next line of management would "eB
a% *sychothera*y and *sychiatric consultation
"% do C+ a"domen
c% do growth hormone stimulation test
d% oral glucose tolerance test
.0. Pt with a &nee ?oint *ain and swelling. As*iration shows a rhom"oid sha*ed
material. #hat is the diagnosis$
a% calcium *yro*hos*hate dihydrate
"% monosodium urate crystals
.2. Pt came "ac& from holidays/ and he is hy*ertensive on thiaGide diuretics. How he
c;o *ain/ swelling/ of right an&le ?oint and fever. #hat is the "est action$
a% an&le as*iration
9
"% serum uric acid level

.7. A *t develo*ed a mallet finger after an in?ury to his index finger. #hat do ex*ect
to find on examination$
a% fixed deformity of the distal *halanx
" I *assive extension of the distal *halanx "ut no active extension
c% resistance to extension
.9. Pt wit 6=C/ what is the most sensitive test
a% AP++
"% P+
c% =H
d% E6P
.<. Post%o* *t continued to "leed des*ite 1> units of *ac&ed AC transfusion.
#hat is the cause of the continued "leeding$
a% 6ilutional throm"ocyto*enia
"% )y*ercalcemia
c% )y*er&alemia
0>. A .0 @! *atient develo*ed a dee*ening ?aundice/ itching/ and light colored stool.
)is "lood showed an elevated level of Al& Phos. #hat is the "est investigation you
can do$
a% :;, a"domen
"% ECP
c% C+ a"domen
d%
PEDIATRICS
01. A mother "rought her 1%year%old child to you with convulsions. )e had a history of
a fe"rile illness recently. +he child was treated with acetamino*hen "ut is still
convulsing. #hat is the next line of management$
a% give diaGe*am P
"% acetaldehyde P
c% =( anti"iotics
d%
02. A child with &nee effusion and "ac& *ain. #hat are you going to do for him$
a% ,acroiliac ?oint x%ray
"% as*iration of the ?oint
0-. A . years old child with stridor/ and res*iratory distress. 1%ray shows thum" sign.
@ou should do all of the following exce*t
a% examination of the throat
"% intu"ate
c% =( anti"iotics
<
d%
0.. Aa"y strictly on mothers mil& for 12 months. @ou would ex*ect him to haveB
a% iron deficiency anemia
"%
00. A child was "orne full term. )e cried s*ontaneously and moved all lim"s
vigorouslyJ he was centrally *in& and "lue in the extremities. )e had cu**it
seccundum. #hat is his a*gar score$
a% -
"% 0
c% 7
d% <
e% 1>
02. A new"orn 7 days old was "rought "y his mother "e cause he is ?aundiced. !n
investigation you find his "illiru"in is 1./ coom"s test negative. #hat is the
diagnosis$
a% "reast mil& ?aundice
"% AA! incom*ati"ility
c% Galactosemia
d%
07. A family came "ac& from Greece/ "rought their child with *allor and fatigue.
=nvestigations show microcytic anemia with "aso*hilic sti**ling. #hat is the "est
management$
a% hemoglo"in electro*horesis
"%
09. All of the following can cause stridor exce*t
a% asthma
0<. =n ne*hrotic syndrome all of the following is ex*ected exce*tB
a% low C-
"% hy*onatremia
c% secondary hy*eraldosteronism
d% hy*erli*idemia
e%
2>. #hich one of the following is the most common trigger of the childhood asthma$
a%
Parental smo&ing
"% house dust
c% rugs
1>
21. =n ne*hrotic syndrome all of the following is true exce*tB
a% 'ow C-
"% hy*er cholesterolemia and hy*ertrygleceridemia
c%
d%
e%
22. A child with ne*hrotic syndrome and ascitis/ develo*ed a"dominal *ain and fever.
#hat is your next line of management$
a% A"dominal *aracentesis
"% :;, a"domen
c% "lood culture
d%
2-. @oung girl "rought "y her mother "ecause she develo*ed tea colored urine/ a lower
lim" edema/ and hy*ertension. #hat is your first investigation$
a% A,! titer
"% urinalysis
c%
d%
2.. A 1>%yr. old dia"etic child was "rought from his school in a comatose state. )e had
a *eculiar odor coming from his mouth/ and had &etone "odies in his urine. #hat
is your initial management$
a% 1> unit fast acting insulin ,C
"% "olus infusion of normal saline
c% =( glucose
d%
e%
20. 1%yr. old child "rought "y his mother for foul odor "ul&y stool with fat dro*lets in
the stool/ the child has "een loosing weight des*ite enormous a**etite. #hat is
your diagnosis$
a% celiac disease
"% cystic fi"rosis
c% hy*erthyroidism
d% ta*e worms
e%
22. Karyoty*ing of a new"orn with hydroce*halus will *ro"a"ly showB
a% mosaiesim
"% translocation
c% *oly*loidy
d% eu*loidy
27. Most common ty*e of 6own8s syndrome is
a% non dys?unction
11
"% translocation
c% mosaiesm
29. A child with *yloric stenosis. #hat is correct regarding his condition$
a% it *ea&s at - months of age
"% visi"le *eristalsis
c% x%ray should "e done
d% develo* meta"olic acidosis
e%

2<. A 12 you "oy "rought "y his mother for difficulty in learning/ lagging "ehind in
school/ fatigue/ and slow growth. #hat is the most *ro"a"le diagnosis$
a% )y*othyroidism
"% congenital mental disorder
c% *anhy*o*itutrism
d% learning disa"ility
7>. A child of a "lac& family is "rought to you with a history of recent tonsillitis/
which he was treated for with trimetho*rim;sulfasexaGole/ one wee& ago. How he
is having fever and his "lood investigation showed )"F90g;l/ and low *latelets.
+he family history is noncontri"utory. #hat is the diagnosis this time$
a% G2P6
"% transient 3a*lastic5 anemia
c% Acute leu&emia
d% s*herocytosis
e% thalasemia
71. A child was "rought lim*ing to your clinic. !n examination he had a *ainful
swelling a"ove his &nee/ and had fever. #hat is the diagnosis$
a% osteomyelitis
"% transient 3toxic5 "ursitis
c% "ursitis
d%

72. A child is "rought to you with 1>D dehydration. +he child loo&s sic& with sun&en
eye"alls. #hat is the initial fluid management$
a% 12>ml;hour H,
"% 12> ml;hour 60#
c% 12>ml;hour 1;-60#2;-H,
d% oral rehydration fluid
e%
7-. A child with hy*ertonic 1>D dehydration. #hat is the main com*lication that
might start with the thera*y$
12
a% arrhythmia8s
"% convulsions
c%
7.. A 12%year%old child com*laining of lower lim" *ain and thigh *ain that wa&ens
him at night and resolve in the morning. #hat is the diagnosis$
a% growing *ain
"% "one malignancy
c% malingering
70. A mother "rought her child who on examination had a syntenal tag. #hat is the
diagnosis$
a% anal fissure
"% hemorrhoids
c% intussusce*tions
72. +he commonest cause of "leeding P in child 1 year old is
a% mec&les diverticulum
"% anal fissure
77. Child with recurrent e*isodes of vomiting and draws his legs u* to his a"domen
and *asses dar& color red stool 6x
i% =ntussusce*tions
79. +he least effective and most associated with side effect vaccine isB
i% Pertussis
ii% 6i*htheria
iii% Measles
7<. Colostrum *rotects against E.Coli due toB
a% =gG
"% =gA
c% =gM
d% =gE
e% =g6
9>. A mother "rings her 2%month%old infant/ telling you that she needs to "reast feed
him every three hours or the child will cry. +he child loo&s normal. #hat are you
going to do$
a% add vitamins to the mother8s mil&
"% tell her it is normal
c% advice her to add to* u* of formula mil&
d% sto* "reast feeding and start iron fortified feeds
91. +he commonest cause of infant mortality is
a% congenital anomalies
"% accidents
1-
c% infections
d%
92. A "a"y "itten "y his *it dog/ what to do$
a% Confine the dog and watch for ra"ies signs and sym*toms
"% ,acrifice the dog with "rain examination for negric "odies
c% (accinate against ra"ies
d% a"ies immunoglo"ulin

9-. !ne of the following is the least common in the differentials of an a"dominal mass
of a 1 year old infantB
a% #ilms tumor
"% neuro"lastoma
c% hydrone*hrosis
d% *olycystic &idney
e% "idudenum
9.. Child "itten "y his neigh"or dog. #hat are you going to do$
i% !"serve the dog for sign and sym*toms of ra"ies
"% sacrifice the dog and examine the "rain for distinctive lesion
c% give antira"ies immunoglo"ulins
90. 'ive attenuated vaccines are the following exce*tB
a% +A
"% @ellow fever
c% Polio
d% MM
e% 6i*htheria
92. A child of an )=( *ositive mother develo*ed recurrent throat infections and later
develo*ed adeno*athy. #hat is your "est investigation to detect the disease$
a% E'=LA
"% PC
c% 'P
d% EAC
97. A child develo*s diarrhea and you sus*ect shigellosis. #hat is the treatment of
choice tell you get the culture re*ort$
a% Am*icilline
"% trimetho*rim;sulfasoxaGole
c% metrnidaGole
d%
1.
99. A child develo*ed a rash over his chest and similar lesions in the "uccal mucosa
and fever. #hat are you going to do$ a% give
him su*erimunoglo"ulins "%
isolate the child from the family
c% inform the *u"lic health authority
d%

9<. )ow can you differentiate "etween an acute tonsillitis and tonsillar a"scess
34uinsy5$
a% trismus
"% *etechia on the *alate
c%
d%
PUBLIC HEALTH
<>. =n the following cohort study two grou*s of sam*les were com*ared. And the
following results o"tained disease no disease
factor 0> <0>
no factor 0
<<0 what is the relative ris& of this study
a% 1>>D
"% 0>D
c% 1>D
d%1>.0D
e%>.1D
<1. A *t that was an alcoholic died of liver cirrhosis. )e was told "y several
ex*erienced doctors that he had an enlarged liver "ut gave him different grades/
e.g. 31-/ 10. 19 cm "elow costal margin5. +his test re*resentsB
a% 'ow validity and high relia"ility
"% high validity and low relia"ility
c% low validity and low relia"ility
d% high validity and high relia"ility
<2. +he commonest ty*e of natural ioniGing radiation comes fromB
a% adon
"% :( light
c% x%ray machines
<-. =n em*loyees of lead refining factory/ what is the "est and most efficient way to
reduce lead *oisoning$
A% wash the hands thoroughly every time they eat.
"% use dis*osa"le outfit
c% wear mas&s
d%
e%
10
<.. Question on chi s4uare study
<0. Question on t%test 3"oth 4uestion are not from the "oo&5
<2. @ou will advice a *erson who is going to an area endemic of cholera toB
a% drin& "oiled water and washed food only
"% tetracycline *ro*hylaxis
<7. =n the industry and eye health
a% the use of welding machines causes irreversi"le corneal o*acity
"% 'aser use leads to s*ecific damage to the macula
c%
d%
<9. Crude "irth rate is
a% the num"er of total child "irth divided over the mid year *o*ulation
"% the num"er of total child "irth divided over the res*ected age grou*
c% the num"er of total child "irth and still "irth over the total num"er of live
Airth
d% the num"er of total live "irth over the num"er of women in the re*roductive
Age grou*

<<. All of the following foods can cause cancer exce*t
a% fat
"% alcohol
c% sodium monoglutamate
1>>. All of the following infections are transmitted via the fecal%oral route exce*tB
a% E"sien Aarr (irus
"% he*atitis A virus
c% ota virus
d% Horwal& virus
e% *olio virus
1>1. Question on the disadvantage of the case control study
a%
"%
c%
1>2. =f a trial carried out and found the relationshi* "etween an intervention and
outcome to have P%value of M>.>0 this will mean
a% there is an association "etween the intervention and the outcome
"% the difference is due to chance alone
c%
d%
1>-. +est is made for readings of AP in elderly *atient and those with increased AP is
termed 3la"eled5 so the term la"eled means
12
a% those are in more serious danger/ and will "e more serious and more
com*liant
1>.. =f you are ma&ing a cross sectional study/ the "est way to ta&e a sam*le is fromB
i% +he general *o*ulation
1>0. ,tudy conducted and result is statistically insignificant/ PF>.. means thatB
i% +he difference could "e due to chance alone
1>2. eha"ilitation after a car M(A meansB
a% *rimary *revention
"% secondary *revention
c% tertiary *revention
1>7. +he function of the royal college of *hysicians in Canada is
a% to act as a *olitical *arty to follow the "enefit of the doctors
"% grants the overseas doctors the licensure to *ractice in Canada
c% to loo& after the *ost%graduate training of the doctors and
it uniformity d%
e%
1>9. A female who had a "reast cancer removed three months ago ant treated with
chemothera*y for the mets. Comes with an acute history of cough and *roductive
of "lood and s*utum/ fever. Pt goes into dee* cyanosis and dies. #hat is the
*rimary cause of her death$
a% *neumonia
"% "reast cancer
c% metastasis
d%
e%
1><. +he commonest occu*ational com*laint is
a% hearing loss
"% contact dermatitis
c% occu*ational cancers
d%
e%
11>. !ccu*ational hearing loss is characteriGed "y
a% worst at high fre4uencies
"% worst at low fre4uencies
c% *rogressive even if ex*osure sto**ed

111. !HE of the following statements is wrong
a% A layer of oGone develo*s near a *hotoco*ying machine in closed area
"%
17
c%
112. +he effect of as"estos and cigarette smo&ing on the lung isB
a% synergistic
"% additive
c%cumulative
d% neutral
e% antagonist
11-. #hat is the mean
a% the average of the total num"er of the values
11.. =n which of the following food does Clostridium "utulinum grow "est$
a% Pac&ed food
"% freeGed food
c% dried food
d%
e%
110. All is irritant to the res*iratory tract exce*t
a% C!
"% H!2
c% ,!2
d% Ammonia
e% Cl
112. ,tudy of M(A mortality is ->D among the drivers with *revious M0yrs ex*erience/
0>D in those with N 1> yr. ex*erience so/ it showed that accidents are more in
those with *revious long ex*erience/ you will disagree with this conclusion
"ecauseB
i% )e did not ta&e the non fatal accidents also among those drivers i.e. no
control grou*
117. #hat is true regarding non%ioniGing radiation$
i% #elding can cause cataract
119. A .2 years old man/ who is o"ese/ has a *ositive family history of C(A his
A*F10>;<0/ smo&es 2> cigarette a day/ has a sedentary life style/ and wears the
seat "uilt 7>D of the time. )e comes to you for a regular chec& u* and as&s you
what can he do to have a healthy life style for the next 1> years. @ou tell himB
a% sto* smo&ing
"% start low dose thiaGide diuretics
c% reduce weight
d% do exercise
19
e% wear the seat "uilt 1>>D of the
time
11<. !ne of the following is not true regarding *lace"o
a% =M *lace"o is more effective than *o *lace"o
12>. #hat is true of the following
a% funds to the hos*ital and doctors will disa**ear over the time
121. =f an autosomal recessive disease is *resent in the community at a rate of 1B.>>>>.
#hat is the *ro"a"ility of any two marring to get a child with the disease$ a%
1B2>> "%
1B.>> c% 1B
2>>>> d% 1B
.>>>>
MEDICINE
122. All of the followings are signs of irrita"le "owel syndrome exce*tB
a% nocturnal diarrhea
"%
12-. =n (on #ill Arand8s disease ty*e = you will find
a% *rolonged "leeding time
"% low fi"rinogen
c% low P++
12.. A female *t *resented to your clinic with history of *rogressive lethargy and
muscle wea&ness/ with *artial *tosis and occasional dys*hagia. !n examination
she was found to have a *roximal muscle wea&ness. )ow are you going to confirm
your diagnosis$
a% muscle "io*sy
"% nerve conduction test
c% tensolin test
120. A 7> yo female *t came with cervical and groin lym*hadeno*athy/ s*lenomegaly/
and raised #AC. ,he gives a history of wt loss and sweating. #hat is the
confirmatory test$
a% lym*h node "io*sy
"% "one marrow as*iration
c%
d%
122. -> yo female *atient with headache *roceeded "y visual distur"ance. #hat is most
li&ely found on examination$
i% ,cintillating scotoma
127. Pt with =A6 what is true
1<
a% extra intestinal features will remain after total colectomy
"% extra intestinal features rarely *resent "efore colonic features
c% ris& of Ca is more in C6 than :C
d% ris& of ?oint disease is more in C6 than :C
129. All of the following organisms cause lung a"scess exce*tB
a% *neumocystis carini
"% sta*haureus
c% &le"siella *neumonia
d% chlamydia
e% myco*lasma
12<. -0 yo Pt with recurrent hemo*tesis and C1 with middle lo"e infiltration. +he
most li&ely diagnosis isB
a% Aronchiectasis
"% lung a"scess
c% chronic "ronchitis
d% em*hysema
1->. Pt with &nown 6M AP 22>;9> what drug you are going to use
i% ACE inhi"itor
1-1. Pt with dys*noea/ hy*otension/ *ulsus *aradoxus/ C1 large glo"ular heart
shadow
i% Cardiac tem*onade
1-2. A 00%yr. old lady came to you with lower extremities "one *ain. +he x%ray showed
a dense "one mass/ and her "lood showed a mar&edly elevated Acid Phos. #hat is
her *ro"lem$
a% Pagets disease
"% osteo*orosis
c% osteosarcoma
d% rheumatoid arthritis
e% Ewing
1--. A *t "rought to the E6 com*laining of sudden onset tearing *ain in his chest
radiating to the "ac&. #hat is correct a"out aortic dissectionB
a% A develo*ing murmur leads to a worst *rognosis
"%
c%
d%
e%

1-.. A .2 year%old male *t with dar& com*lexion develo*ed glucosuria/ *al*a"le liver
edge. #hich of the following will hel* you in your diagnosis$
a% serum ferritin level
"% *lasma cerulo*lasmin
2>
c% al*ha one antitry*sin

1-0. +he following drawing of a C1 and wrist *ain is for a *t com*laining of a
history of wt loss and fatigue for the *ast < months. #hat is the diagnosis$
a% "roncogenic carcinoma with hy*ertro*hic
*ulmonary osteoarthro*athy "% rheumatoid disease with rheumatoid
nodule c% osteogenic carcinoma
d% osteosarcoma
e%
1-2. =nitial treatment of acne volgaris is
a% to*ical "enGyl *eroxide
"% to*ical steroids
c% to*ical isoretenoin
d%

1-7. #hich of the following is the "est investigation for hemolytic anemia
a% reticulocyte count
"% E,
1-9. Pt comes with an intense itch in the hand and "etween the fingers. #hat is the
cause$
a% sca"ies
"%
1-<. A *t came with a *ain in the lower leg. An x%ray was ta&en and showed a sun"urst
ty*e fracture of the ti"ia. #hat is the cause of it$
a% !steoid osteoma
"% osteosarcoma
c% rheumatoid arthritis
1.>. A *t has "een a chronic smo&er and now he has CA!6 with *in& *uffer *icture.
#hat is the "est long%term management$
a% home oxygen
"% CPAP
c%
1.1. A female *t 02%yr. old right%handed develo*ed sudden onset headache and then
could not find the words she wants to say. #here is the lesion in her "rain$
a% 'eft frontal lo"e
"% left *arietal lo"e
c% left tem*oral lo"e
d% right tem*oral lo"e
21
e% right frontal lo"e

1.2. A 7>%yr old lady/ a &nown case of '() and A6 insufficiency/ came with AP of
9>;.> and no *al*a"le *ulseJ her general condition is good. #hat is your initial
management$
a% =( fluids
"% atro*ine
c% adrenaline
d% digoxin
e%
1.-. Pt 2 days *ost M= in the CC:/ develo*ed tachycardia/ sweating/ and constricted
*u*ils. #hat is the diagnosis$ a%
mor*hine toxicity
"%
c%
d%
1... Pt 2 days *ost M= in the CC: develo*s agitation/ hallucinations/ and delirium.
#hat is this management this time$
a% Mor*hine sulfate
"% halo*eridol
c% diaGe*am

1.0. Pt with *arasternal heave/ loud ,1/ and a diastolic murmur in the a*ex of the heart.
#hat is the diagnosis$
a% Mitral stenosis
"% aortic stenosis
c%
d%
e%
1.2. A 2-%yr. old girl is on *henytoin ->>mg and com*liant/ "ut the seiGures are not
controlled. #hat are you going to do for her$
a% increase the dose to .>>mg
"% change to another drug
c% add another drug
d%
e%
1.7. A young male develo*s 2 e*isodes of synco*e after weight lifting exercises. )e
tells you that his father died at the age of -.. #hat is the "est investigation you
would do$
a% Echocardiogram
"% C1
22
c% ECG
d%
e%
1.9. =n the exercise stress test/ all of the following are indication to sto* the test exce*tB
a% chest *ain
"% ,+ elevation
c%
d%
e%
1.<. A 21 yr. old girl comes to you com*laining of *al*itation of sudden onset/ her
heart rate is N10>/ other wise she is normal. #hat is your management$
a% give her *ro*ranolol
"% try vagal stimulation
c%
d%
10>. A *t comes to you with sudden loss of vision in the left eye for few minutesJ it is
descri"ed as a curtain coming down. ,imilar com*lete loss of vision occurred in
the left eye in the *ast "ut returned to normal later. #hat is the affected artery$
a% left internal carotid
"% right internal carotid
c% left *osterior cere"ellar artery
d% left *osterior cere"ral artery
e%
101. Pt *resents to you with onset of s*linter hemorrhage in his fingernails/ he gives
you a history of "eing unwell over the last *eriod with fatigue/ wt loose and fever.
!n examination he had a murmur in the heart. #hat is the "est investigation you
can do to confirm infective endocarditis$
a% Alood culture
"% echocardiogram
c%
d%
102. Pt &nown hemo*hiliac was receiving *latelet infusion develo*ed hy*otension.
#hat are you going to do$
a% sto* the transfusion
"% slow down the transfusion
c% give =( antihistamine
d%
e%
2-
10-. @ou are called to see a 7> yr. old lady who had AP 9>;.> and C(P 17 cm)2!
what is your diagnosis$
a% ,e*tic shoc&
"% Cardiogenic shoc&
c% hy*ovolemic shoc&
d%
e%
10.. +he antidote to acetamino*hen is
a% H%acetyl cystine
"% nalaxone
100. A young adult was recovered from a "urning "uilding and "rought to the
emergency de*artment/ semi%comatosed and in res*irator distress. !n examination
his nasal mucosa loo&ed "lac&. #hat do you ex*ect his "lood gases to have$
a% low Pa!2
"% )igh concentration of car"oxyhemoglo"in
c% high level of PaC!2
d%
e%
102. A grou* of children had a meal 2 hours later all of them develo*ed a"dominal *ain
and vomited. +hey were sic& for . hours and then settled. #hat is the organism
that caused this$
a% sta*haurius
"% E.coli
c% com*ylo"acter ?e?ini
d% clostridium "utulinium
e%

107. =n which of the following age grou*s is the Canadian *o*ulation ex*anding$
a% 1%0
"% 0%10
c% 10%->
d% ->%00
e% a"ove 2>
109. A *t "rought to the emergency de*artment/ after staying in the cold and develo*ing
frost"ite in his feet. #hat are you going to do for him$
A% re%worm ra*idly
"% cover with clean dry cloth
2.
PSYCHIATRY
10<. A 21 years old girl came "ac& from a *arty and started acting strangely and
indecently/ and was ver"ally a"usive. ,he was tachycardic and had muscle rigidity.
#hat is the diagnosis$
a% Cocaine ingestion
"% canna"is intoxication
c% am*hetamine
d%',6
e% PCA
12>. Psycho stimulants are used in which of the following disorders$
a% attention deficit hy*eractivity disorder
"% tics disorder
c% childhood schiGo*hrenia
121. A manic *t that is on 'ithium treatment/ develo*ed tremor and tahycardia. #hat is
your management$
a% *ro*ranolol
"% "enGotro*ine
c% amitri*talline
d%

122. 6rug that is most associated with amnesia isB
a% Phenytoin
"% 'ithium
c% 'oraGe*am
d%
e%
12-. =n *ost*artum *sychosis/ all of the following is true exce*tB
a% its left with residual effect
"%
12.. A 22 years old female comes to as& you a"out )untington8s chorea. )er father has
it/ her *aternal grand father had it and her *aternal anti had it. #hat is her chance
of develo*ing it$
a% 1>>D
"% 0>D
c% 20D
d% 12.0D
e% she will not develo* it
20
120. A *atient who is a friend of yours has recently divorced his wife. How he comes
telling you that he is going to &ill his ex%wife and her "oyfriend. #hat are you
going to do$
a% tell him you will "e o"liged to inform his wife if he does not change his
mind
"% write to the wife telling the situation
c% inform the *olice
d% discuss it with your lawyer and then document it in the *t8s chart

122. #hat do you see in gender a"normalities$
a% cross%dressing in childhood.
"%
127. +he defense mechanism in *ho"ia isB
a% dis*lacement
"% dissociation
c% intellectualiGation
d% association
e%
129. All of the following drugs can cause mania exce*t
a% corticosteroids
"% am*hetamines
c% ami*ramine
d% car"amaGe*ine
e% cimitidine
12<. All of the following drugs cause de*ression exce*t
a% ce*halos*orins
"% steroids
c%
d%
17>. =n alcohol toxicity all of the following occur exce*tB
a% tongue tremor
"% "radycardia
c%
171. +he difference "etween schiGo*hrenia and mania isB
a% +hought "roadcasting
"%
c%

172. =n "ulimia nervosa all of the following occur exce*tB
a% dental *ro"lems
"% *arotid gland enlargement
22
c% meta"olic acidosis
d%
17-. A *t "rought "y her friends/ saying that her "oss cannot tolerate her anymore. Eor
the *ast 2 years she has "een feeling laGy and active "ut now suddenly she realiGed
what she has missed/ and she is ta&ing too much wor& that she cant finish and
increasingly ex*ressed wrong ?udgments and "een a"using the drin&s. ,he feels
normal and does not understand why her "oss wants a doctor to see her. #hat is
the diagnosis$
a% de*ressive e*isodes
"% cyclothymia
17.. A *t "een feeling low for the *ast . years and has lost all interest in life and does
not leave home exce*t rarely and doesn8t want to see anyone. #hat is the
diagnosis$
a% dysthymia
"%
170. #hat is the treatment of *ost traumatic stress syndrome
a% *sychothera*y
"% *sychoanalysis
c% drug thera*y
172. Pt with tachycardia/ sweating/ and dilated *u*il. #hat is the li&ely drug to cause
it$
a% 6isulfaram
"% *hencyclidine
c% "enGotro*ine
d%
177. ,chool *ho"ia in young child
i% eturn immediately to school and family disclosure
179. AlGheimer8s dementia/ what is true$
i% A*hasia/ agnosia/ a*raxia
17<. -0 yo *t found wandering in the "ac&yard/ all *ossi"le etiologies exce*tB
i% AlGheimer dementia
19>. Pt treated for *sycho stimulant toxicity. #hich of the following you are going to
do to minimiGe com*lications
i% Eollow u* su**ort grou* with daily monitoring
191. Adolescent female left "y her "oyfriendJ she swallowed 2> ta"s of (alium. After
you did gastric lavage/ you assessed her she is somewhat well with feelings of
remorse/ what will you do next$
27
i% 6ischarge her home
ii% !"serve for 2. hrs
iii% Consent and admit her
192. Most *rominent sign in o*ioid toxicity is
i% Constricted *u*il
19-. Eemale *atient *resented with ataxia and "ilateral hand wea&ness/ neurological
exam is normal/ and she has recently "een divorced. 6x
i% Conversion disorder
19.. < yo girl with difficulty in learning/ de*ressed/ cannot recogniGe colorful things
exce*t one color and she "ecomes ha**y only when she understands you. #hat is
her *ro"lem$
ii% A6)6
iii% M
iv% 'earning disorder
190. Pt with hy*ertension/ memory loss/ neurological sym*toms/ 61
i% Multi%infarct dementia
192. -0 yo *t *resented with decreased concentration/ slowness of thin&ing a*athy/
socially withdrawn/ short term memory loss with difficulty in learning new
information then develo*ed seiGure. #hat is the diagnosis$
i% A=6,%dementia com*lex
197. Alcoholic *t found in the *ar& "ehind a "ar with confusion/ restlessness and
refusing any instruction for exam. #hat is your management$
i% =( glucose
"% =( chlordiaGe*oxide
199. Pt alcoholic with &nown liver cirrhosis *resented with confusion/ restlessness/
generaliGed ecchymosis and characteristic "reath odor. #hat is the diagnosis$
i% )e*atic ence*alo*athy
"% #erne&es ence*halo*athy
c% Korsa&of ence*halo*athy
19<. =n schiGo*hrenia all of the following is true exce*t
i% Arief recurrent *sychotic attac&s
1<>. #hich of the following is a characteristic of *ost%traumatic stress disorder$
i% )y*er vigilance
1<1. 6efense mechanism of "orderline *ersonality disorder is
i% ,*litting
29
1<2. Child with autistic disorder/ which of the following you can tell the mother that/
indicates favora"le outcome$
i% 6ecreased irrita"ility
ii% =m*rovement in language communication s&ills
iii% =ncreased self awareness
d% im*rovement in social imitative *lay
1<-. A &nown alcoholic man *resented with confusion/ ataxia/ ina"ility to a"duct "oth
eyes with nystagmus/ liver edge *al*a"le at - cm "elow costal margin/ yellowish
discoloration of the s&in/ sclera with )x of +A/ sy*hilis/ (6' *ositive in many
occasions. #hat is the most li&ely diagnosis$
i% #erni&e8s ence*halo*athy
1<.. Question on the inner feeling of a *erson and how it affects the exterior 3cant
remem"er it well5
a% ego
"% su*er ego
c% self
d%
e%
1<0. @ou are called to the *olice to interview a 02 year%old female *atient/ who has
"een sending the *olice letters that she sees flying saucers. #hen you as& her
hus"and he denies seeing any thing. )ow are you going to manage her$
a% admit her and start anti*sychotic thera*y
"% tell her that you "elieve her
c%

OBSTETRIC & GYNECOLOGY
1<2. A *regnant lady 2>%wee& gestation came into contact with a child who has ru"ella.
#hat are you going to do for her$
a% do ru"ella titer
"% advice her re thera*eutic a"ortion
c% give her ru"ella vaccine
d% give her ru"ella anti"odies
c%
1<7. A *regnant lady in la"or has her legs hy*er flexed at the hi* ?oint. #hat is the
associated condition$
a% shoulder dystocia
"% "reech *resentation

1<9. #hat is the commonest com*lication of dermoid cyst$
a% torsion
2<
"% ru*ture
c% hemorrhage
d%
1<<. Eemale *atient with high levels of *rolactin. #hich one of the following hormones
you are going to chec&$
i% +,)
2>>. All of the following is true regarding "acterial vaginosis exce*tB
a% green color discharge
"% foul fishy smelling when K!) is added
c% *aucity of lacto"acilli
d%

2>1. #here is the "est *lace to ta&e a *a* smear$
a% endocervix
"% ectocervix
c% endo and ectocervix
d% ectocervix and *osterior vaginal wall
e% endocervix and *osterior vaginal wall
2>2. 2< year%old *resented with history of "leeding after 1
st
cycle/ which was 0 wee&s
ago. !n examination she had normal findings and a closed os/ and her u;s showed
an em*ty uterus with a cystic mass in the ovary. +here are no signs of *regnancy.
#hat is the diagnosis$ a%
com*lete a"ortion with leuteal cyst
"% missed a"ortion with rt follicular cyst
c% hydatidiforme mole with thecal cyst
d% ecto*ic *regnancy
2>-. Primigravida in la"or/ "a"y at > station/ ce*halous *resentation and occi*eto%
transverse. Cervix is . cm dilated since 2 hours with effacement a"out ->D.
:terine contraction every 0%1> mins and lasting for 10 secs. #hat is the diagnosis$
a% normal la"or
"% o"structed la"or
c% ce*halo*elvic dis*ro*ortion
d% mal*osition of the fetus
2>.. A woman who is a grand multi*ara and is in full term and has a transverse lie. ,he
had her mem"ranes ru*tured. #hat are you going to do$
a% attem*t external version
"% *re*are for immediate C%section
c% allow to *rogress and s*ontaneous correction
d%
2>0. All of the following can cause *olyhydramios exce*tB
a% renal agenesis
->
"%

2>2. 21 years old girl was on the "irth control *ill and sto**ed it five wee&s ago
*resented to you with secondary amenorrhea. )er urine AhCG was negative 2 days
ago. #hat are going to do for her$
a% *ost meno*ausal lactogen
"% serum AhCG
c% *rogesterone withdrawal "leed
d% +,)
e% estrogen level

2>7. =n uterine cancer all of the following is a ris& factor exce*tB
a% smo&ing history
"% uno**osed estrogen thera*y
c% multi*arity
d%
e%
2>9. Pa* smear with moderate to severe dys*lasia. #hat to do next$
a%col*osco*ic examination
"% cone "io*sy
c% laser a"lation of the cervix
2><. All of the following cause increased levels of gonadotro*hins exce*tB
ii% Ailateral dermoid cyst
21>. Pt with infertility/ her hus"and had three children from other wife/ she had a
diagnosis of *erforated a**endix o*erated u*on. ,he has normal ovulation. #hat
is the "est test for diagnosis$
i% 6aily chart of AA+
ii% endometrial "io*sy
iii% hysterosal*ingogram
iv% Post coital test
211. A lady with =:C6 develo*ed "ilateral lower a"dominal *ain. #ith tenderness of
the cervix on examination. #hat is the diagnosis$
a% sal*ingitits
"% ecto*ic
c% a**endicitis

212. +he commonest cause of late *ost*artum hemorrhage isB
a% uterine atoney
"% uterine involution
c% s*ontaneous ru*ture of vaginal hematoma
d%
-1
21-. A lady who is "reast%feeding comes to you with *ainful swelling in her "reast that
is hot and tender to touch. #hat is correct regarding her condition$
a% its caused "y a gram negative "acteria
"% incision and drainage is mandatory
c% it commonly occur 2%. wee&s *ost*artum
d% fever N-9
e%
21.. A lady who is *regnant at 2> wee& gestation/ the fetus will "e at the level ofB
a% at the um"ilicus
"% *u"ic sym*hesis
c% xe*hoid *rocess
d% a"ove the um"ilicus
210. A .-%yr. old woman comes to you com*laining of having "leeding from her vagina
every time she has sex with her hus"and/ which can "e *ainful some times. !n
examination you find an ulcerated area on her cervix. #hat is the diagnosis$
a% sy*hilitic ulcer
"% cervical carcinoma
c% cervecitis
d% cervical carcinoma insitu
e%
212. A *regnant lady with a history of *revious C,/ during delivery she had her
mem"ranes ru*tured 3"y amniotomy5 a gush of "lood a**eared and minutes later the
fetal monitoring showed signs of fetal hy*oxia/ and the mother started getting
tachycardia. #hat is the li&ely diagnosis$
a% *lacenta acrreta
"% *lacenta *revia
c%
d%
217. Pt with mass "ehind the ni**le and the ni**le "leeds when it is com*ressed. #hat
is the li&ely diagnosis
i% =ntraductal *a*illoma
219. A *regnant lady who is in 2>%w&. gestation came to you and found to have the
*lacenta *artially covering the internal os. #hat are you going to advice the
mother to do$
a% com*lete "ed rest
"% tell her she will need C, later
c% re*eat :;, at 29 w&.
d%
e%
21<. A *regnant lady develo*s *ain in her flan&/ fever and chills. #hat is the diagnosis$
a%
-2
*yelone*hritis "% :+=
c%
d%
22>. A full term *rimigravida is in la"or and had a *rotracted latent *hase. #hich one
of the following drawing corres*onds with it$
d% 'ine365
cevical
dialtation
5cm5
> 2 . 2 9 1> 12 1. 12 19 2> 22 2. 22 29 -> -2 -. -2
'a"or3hours5
221. A s&etch of tem*erature chart 3of the ovulation5/ there is no mid cycle surge in
"asal "ody tem* of >.0 degree to corres*ond with ovulation/ for a female who is
trying to conceive for the last < months. ,he did manage to conceive so far.
'oo&ing at her tem* chart you will tell her thatB
a% she has an anovulatory cycle
"% she has ovulatory cycle
6ay 1.
222. =n the course of sexual excitement you will find all of the following exce*tB a%
uterine contractions
"% ni**le erection
c% increased vaginal secretions
22-. A *regnant lady with fi"roid/ she is in her second trimester/ what is the "est
management$
--
1>
9
2
.
2
A A
C
6
E
a% wait till after the delivery
"% do fi"roid lum*ectomy
22.. +he commonest cause of hirsutism isB a%
constitutional "%
*olycystic ovarian disease
c% congenital adrenal hy*er*lasia
CAHA6=AH E(A':A+=HG E1AM
MAC) 1<<9
-.
1% A'' !E +)E E!''!#=HG 6:G, AE 6EPE,,AH+, E1CEP+B
A5 A%A'!CKE,
A5 !A' C!H+ACEP+=(E P='',
C5 C'!HA6=HE
65 CEP)A'!,P!=H,
E5 C!+=C!,+E!=6,
2% PE(A'EHCE !E AH =HEEC+=!:, 6=,EA,E =, AEEEC+E6 A@ A'' E1CEP+B
a5 mood of transmission
-% 9. @! female has mild chronic confusion/ C;! u**er a"dominal *ain when eating/
wa&es her at night. ,he smo&es 1 *ac&et of cigarette daily. ,he is ta&ing H,A=6, for
osteoarthritis. All of the following are *art of the treatment exce*tB
A5 ,:C!'EA+E
"5 )2%"loc&ers
C5 AH+AC=6
.% A'' CAH !CC: =H E1C=+EMEH+ P)A,E !E H!MA' ,E1:A' =H+EC!:,E E1CEP+B
a5 uterine contraction
0% recurrent a"ortion associated with all of the following exce*tB
A5 )EM!)AG=C C!P:, 'E:+EA' C@,+
2% +)E P!+EH+=A' @EA, !E '=EE '!,+ ME+)!6 =, A :,EE:' #A@ !E '!!K=HG A+ M!+A'=+@ E!
A'' +)E E!''!#=HG EA,!H, E1CEP+B
a5 it estimates the average time a *erson would have lived if he had not died
*rematurely
7% the following su"stances may increase the effect of "enGodiaGe*ine exce*tB
A5 C=ME+6=HE
"5 alcohol
C5 AH+AC=6
d5 disulfiram
E5 E,+E!GEH
9% +)E E!''!#=HG =, H!MA' =H PEGHAHC@ E1CEP+B
a5 a**earance of 2%cm mo"ile mass in the "reast
A5 6E(E'!PMEH+ !E +A'AHGEC+A,=A,
<% . M!H+), =HEAH+ #=+) A!HC)=!'=+=, A'' !E +)E E!''!#=HG ,)!:'6 AE (=E#E6 #=+)
C!HCEH E1CEP+B
a5 res*iratory rate of 2>;min
A5 E6:CE6 !A' =H+AKE
c5 h;o *revious hyaline mem"rane disease
-0
1>% the following contraindicated to continued e*idural anesthesia exce*tB
A5 PEMA+:E 'AA! A+ -0 #K.
"5 hemorrhage diathesis
C5 ':MAA 6=,C )H=A+=!H <
d5 *latelets count M 2>x1>
E5 =HEEC+E6 P='!H=6A' ,=H:,
11% =H=+=A' MAHAGEMEH+ !E A=P!'A PA+=EH+ =H MAH=C P)A,E =HC':6E A'' !E +)E E!''!#=HG
E1CEP+B
a5 Heurole*tics
A5 AH+=6EPE,,AH+,
c5 'ithium car"onate
65 EC+
12% C)='6 PE,EH+, #=+) ,E(E PA''!. A'' #!:'6 'EA6 +! ,:,PEC+ AC:+E 'E:KEM=A E1CEP+B
a5 neutro*enia
A5 ,EP+=CEM=A
c5 chest infection
65 )EM!G'!A=H:=A
1-% all are characteristics of Chinese restaurant syndrome
a5 "urning sensation *articularly in the u**er a"domen
"5 sym*toms usually "egin 2>%-> mins after eating
1.% -7 yo male *resented with *ain and "lurring of vision in one eye with vesicle
visi"le in the cornea. All of the following are management of her*etic eye infection
exce*t
a5 to*ical steroid
10% Poor com*liance with thera*eutic regime is a well &nown *henomenon among
some *t. Eactors &nown to "e associated with *oor com*liance include all exce*t
a5 drug induced side effects
"5 alcohol a"use
c5 social *ro"lems
d5 com*lex treatment regimes
e5 fear of addiction
12% in Canada low income is associated with increase ris& of all exce*tB
a5 low "irth wt
"5 infant mortality
c5 varecella
d5 fire death
e5 social "ehavior *ro"lems
-2
17% all of the following statements concerning )e*%A vaccine are correct exce*tB
a5 it is of no value in chronic )A( carrier
"5 it *revent acute he*%" infection
c5 more efficient if given in the deltoid muscle
19% .> yo male has h;o schiGo*hrenia treated "y *henothiaGines. Presented with
chorioathetoid movements of the hand and lim"s/ 6x isB
a5 tardive dys&inesia
1<% least res*iratory infection due to Myco*lasma *neumonia is B
a5 ra*idly *rogressive *neumonia associated with cyanosis
2>% .. ys male *resented - w&. after "rea&down of marriage c;o low mood/ nausea/
anorexia/ insomnia/ and low self%esteem. )is mother and sister are treated for
de*ression. which one of the following in the hx suggests that he will "enefit from
tricyclyc antide*ressantsB
a5 mother and sister treated from de*ression
21% com*lication of ?uvenile rheumatoid arthritis isB
a5 iridocyclitis
22% in M, the neurological sign least li&ely to occur is
a5 anosmia
2-% high s*ecificity of :+= in infants
a5 N 1>#AC in high *ower microsco*y
"5 Ove nitrate in the urine di*stic& test
2.% 2> yo male with hematocasia no hematemesis. 2;12 ago had gastrosco*y/ Aa%meal/
Aa%enema and colonosco*y/ all were normal. 6x
a5 angiodys*lasia
20% 2>yo male - days *ost vulgration of "ladder tumor develo*ed fever
a5 se*ticemia
20% Pt with avoidant *ersonality disorder
a5 *reoccu*ation with details to the extent that wor&ing goes uncom*leted
22% "loody diarrhea *ost radiothera*y for ca cervix
a5 *roctitis
27% 11yrs "oy "ecame anxious/ few w&. ago mother was admitted to the hos*ital.
Child has fear that she might die/ and he is refusing school. 61
a5 over anxious disorder
-7
29% all of the following conditions are included in the 66E of anxiety disorder exce*tB
a5 schiGo*hrenia
"5 hy*othyroidism
c5 *anic disorder
d5 dm
2<% the indication for non stress test include all of the following exce*t
a5 Gestation of .> w&.
"5 dm
c5 chronic hy*ertension
d5 unex*lained *revious still "irth
->% ..yo female with urinary incontinence routine evaluation include all exce*t
a5 &ee*ing urinary diary
"5 voiding cystourethrogram
-1% decreased incidence of C(A is achieved "yB
a5 decreased levels of elevated "lood *ressure
"5 using as*irin
-2% 20 yo male with enlarged testes discovered after mild trauma. =t is firm and
*ainlessJ e*ididymis is felt se*arately from the mass. Ho transillumination. 6x is
a5 varicocele
"5 chronic e*idedemitis
c5 testicular tumor
--% the following indicate foreign "ody in the vagina exce*t
a5 "ad smell
"5 *ainful intercourse
c5 s*otting of "lood
d5 fever
-.% *t with signs and sym*toms of *elvic inflammatory disease x is
a5 oral doxycycline O =M ceftriaxon
"5 admit and start oral doxycycline O =( ceftriaxon
-0% di*htheria *harengitis initial treatment include
a5 6e*htheric antitoxins
-2% .. yo male seeing you wee&ly since his wife left him 1;12 ago. )e informs you
that he intends to &ill his ex%wife and her "oyfriend what will you do$
a5 in form his ex%wife and her "oyfriend
"5 inform your lawyer
c5 document this event in his case record
-9
-7% all of these hormones can "e seen in a *remigravida exce*t
a5 estrogen
"5 *rogesterone
c5 *rolactin
-9% Pt is *ost la*arotomy he develo*s "leeding from the site of the incision *ost o*/
his P++ is high what is the treatment$
a5 *latelet transfusion EEP
-<% which of the following investigation is diagnostic in an infant with (it%K
deficiencyB
a5 "leeding time
"5 *rothrom"in time
c5 =H
.>% ,ecretary has a mass of 2x-cm at the anal cleft 7.0cm away from the anus 6x
a5 infected *ilonidal sinus
.1% *t has ->D 2
nd
degree "urn of the left side of the "ody. All of the following
included in the treatment exce*tB
a5 whole "lood transfusion
.2% *t *ost electrical "urn from hand to shoulder x isB
a5 de"readement and fasciotomy
.-% #hat is the difference "etween *suedodementia and dementia
..% *t .days *ost a**endectomy c;o consti*ation since first *ost o* day 6x isB
a5 *aralytic illius
"5 small "owel o"struction
c5 fecal im*action
.0% *t *ost 6PC -%days later she develo*s high tem*erature which did not res*ond to
*enicillin
a5 retro*eritoneal a"scess
.2% *t c;o limitation of shoulder movement with *ain at 2>%12> degrees no h;o trauma
6x
a5 "ice*s tendenitis
"5 calcification at the rotator cuff muscles
c5 calcification of the ?oint
-<
.7% Pt has ureteric stone measuring >.0cm in diameter/ =(P showed mild
hydrone*hrosis and some dye could "e seen in the ureter what to do.
a5 o"servation
"5 ne*hrostomy
c5 transurethral lethotomy
.9% 19 yo lady comes to you c;o menorrhagia she uses .> di**ers and it last for 0%
days. )er *eriod is of -> days duration what will you do$
a5 reassurance
.<% 00%yo male came with his wife to you with com*lain that he has im*otence for
2;12. )e is alcoholic has erection in early morning and when he see sexual
magaGines. #hat is your management$
a5 as& for glucose levels
"5 marital thera*y
0>% !"ese male comes to you c;o severe 'ow Aac& Pain. )e is alcoholic/ when you
examine him he was screaming s*ecially with the strait leg%raising test. Aut you were
a"le to raise it fully. @our 6x is$
a5 Malingering *t
01% A 70 yo man came to your office "ecause he is having difficulty hearing in "oth of
his ears/ you examine him and find sensory neural hearing loss. +he most *ro"a"le
diagnosis is$
a5 incars disease
"5 *res"ycosis
c5 ottosclerosis
02% 1%ray of a child showing widening of the ends of "oth ti"iae and femurs and
cu**ing. +he family "rought their 2 yo girl "ecause she hasn8t wal&ed yet. #hat is
you diagnosis$
a5 (itamin C deficiency
"5 Achondro*lasia
c5 ic&ets
d5 (itamin A deficiency
0-% 1%ray of a new"orn child showing shifting of the mediastinum to the right side
and hy*erleuscent left lung. #hat is your a**ro*riate management$
a5 =ntu"ation
"5 Haso*harengeal suction
c5 Heedle as*iration
d5 Aronchosco*y
0.% =ns*iratory and ex*iratory chest x%ray of a child showing a hy*erleuscent left lung/
which fails to decrease in ex*ansion on ex*iration. +he diagnosis is B
a5 Aall valve in the right lung
"5 Aall valve in the left lung
.>
c5 ,te* valve in the right lung
d5 ,te* valve in the left lung
00% Picture of 1%year old "oy showing evidence of scalding "urns of "oth legs
extending from the toes to the &nees. ,ome fi"rotic "ands are evident. #hat is the
cause$
a5 Crawling over hot surface
"5 Electrical "urns
c5 Congenital
d5 Eorceful immersion in scalding water
02% Picture of "a"y8s face showing e*icanthal folds and u*slanting of the *al*e"ral
fissure. A mother "rought her child to you for evaluation. ,he says he is flo**y. +he
"est test for diagnosis is B
a5 !"tain family *edigree
"5 Karyoty*ing
c5 1%ray of the scull
07% A mother "rought her child "ecause he is very *ale. CAC shows microcytic
anemia/ "lood film showed target cells and "aso*hilic sti**ling. @ou o"tain CAC of
father and mother. +he father test showed mild microcytosis and hy*ocromia. +he
mother showed the same as the child. +he next test you should do is B
a5 "one x%ray for lead mar&ings
"5 hemoglo"in electro*horesis
09% Picture of "ig toe of a woman showing mar&ed erosion of the nail "ed/ which is no
longer evident. ,he first noticed an ulcer under her "ig toe a year ago and has
*rogressed to the *resent state. #hat would you do$
a5 6e"ridement and anti"iotic cover
"5 Give anti"iotic and see the *atient in a follow u*
c5 Aio*sy the lesion
d5 Examine the toe for *ossi"le gangrene
e5 Examine a swa" under microsco*e for actinomycetes
0<% A woman is referred to you "y the local *olice for evaluation. ,he has sent them
two letters saying that flying saucers have "een landing in her farmyard. @ou
interview her hus"and and he denies seeing anything. #hat will "e your a**roach$
a5 Council her and "egin anti*sychotic thera*y
"5 Advice her hus"and to dissolve the medications in her drin&s
c5 Allow her to ex*lain her theories to you in details
d5 +ell her that you "elieve her
2>% @ou are called to the *olice station to interview a *risoner who had sexually
assaulted a young "oy. )e refuses to answer any of the *olice 4uestioning other than
.1
his name and age. @ou as& him how many legs have a horse and he says five. #hat is
your diagnosis
a5 schiGo*hrenia
"5 "i*olar disorder
c5 "orderline *ersonality disorder
d5 malingering
e5 mental retardation
21% A recently divorced man has "een seeing you for several wee&s for concealing. )e
informs you that he is going to &ill his x%wife and her "oyfriend. @ou should
a5 =nform the local authorities
"5 #rite to the x%wife informing her that she is in danger
c5 +ry to tal& him out of it and document your effort in his file
d5 Get the o*inion of another colleague
22% A 22 yo man came to you com*laining of headache and redness in "oth eyes and
stic&y discharge. !n examination you find his AP 170;7> mm)g. @our management
is
a5 ,end swa" from "oth eyes/ and start local anti"iotic thera*y and arrange
follow u* in the clinic.
"5 Perform tonometry on "oth eyes
c5 Arrange for evaluation of his hy*ertension
2-% A young "oy was found "ehind a "ar/ delirious and agitated. Examination showed
hy*ertension/ tachycardia/ muscle rigidity and loss of sensation to *ainful stimuli.
+he most li&ely diagnosis is
a5 ',6
"5 Alcohol
c5 )eroin
d5 Cocaine
e5 PCP
2.% +he mother of a . yo "oy "rings him to you for evaluation. )e has already "een
using 2%- word sentences/ "ut recently has reverted to one word only and is
stammering constantly. )e must re*eat words many times "efore he can com*lete a
sentence. Eor your evaluation you need to
a5 Council the *arents
"5 ,end the child for EEG
c5 !rder an EH+ evaluation
d5 Get a *sychiatric evaluation
20% After com*lete delivery of twin =/ the *resenting *art of the twin == is guided to the
*elvis and mem"ranes are ru*tured. How the mother has no uterine contractions/ the
cervix is 9 cm dilated and the station is 1. )ow will you manage this *atient
a5 Eorce*s extraction
"5 Cesarean section
.2
c5 ,tart oxytocin
22% the first sign of hearing loss in a *erson wor&ing in noisy environment is
a5 *ain in "oth ears
"5 continuos tennitis
c5 a notch at .>>> )G
d5 una"le to her sounds of high fre4uencies

27% A child was "rought to the E6 with dehydration/ and you treated him with 0D
dextrose for one hour. +hen the child convulses and you find he has low Ha in the
"lood. #hich of the following is the correct management$
a5 give hy*ertonic saline
"5 give =( mannitol
c5 give 0>D dextrose
d5 give hy*ertonic saline and steroids
29% A 7%year%old dia"etic "oy was found unconscious in the school *layground. +he
first thing you should do when he is in the E is
a5 await for "lood glucose level "efore starting insulin
"5 infuse 0>ml 1>D dextrose immediately
c5 give =( insulin
d5 *erform a finger *ric& test for glucose level
2<% Picture of a 0%year%old "oy. )is li*s have multi*le ulcers and whitish lesions. )e
was found in the &itchen and a "ottle of detergent was s*ilt on the floor. #hat is the
"est management$
a5 induce vomiting
"5 give active charcoal and then do endosco*y
7>% A 02%year%old man com*laining of two e*isodes of hematocaGia came to your
office. !n examination he is healthy/ has a mild microcytic anemia. :**er G= studies
was all normal/ re*eated stool analysis and culture was negative for occult "lood and
*arasites. A dou"le contrast enema was negative for *oly*s. #hat is the most li&ely
cause of his "leeding
a5 a "leeding disorder
"5 angiodys*lasia
c5 diverticulitis
71% =mmediately after delivering vaginally of a .>>>%g infant/ the mother "leeds
10>>ml of "lood *er vagina. +he most li&ely cause of this "leed is
a5 Cervical laceration
"5 etained *lacenta
c5 :terine atony
.-
72% assisted delivery of an infant with vacuum extraction increases his ris& of
develo*ing
a5 )y*oglycemia
"5 Arain edema
c5 Ce*halohematoma
7-% A -.%year%old came to you with a confluent vesiculo*a*ular rash over the t
chee& and extending over the "ridge of the nose. #hich statement is true regarding
this lesion
a5 =ts margins are well demarcated
"5 )e will have vesicles in the roof of the mouth
c5 )e will have vesicles in the sinus mucosa
7.% A *regnant woman at 29%wee& gestation re*orts to you that her 2%year%old son
develo*ed measles. @our action should "e
a5 advice for a"ortion
"5 se*arate the child from the mother
c5 *erform anti"ody tests on the mother for rising titers
d5 examine the child for anti"odies
70% A -1%year%old woman with e*ile*sy is controlled on *henytoin. ,he "ecomes
*regnant. @our management of her is
a5 change her to val*roate
"5 sto* anticonvulsant thera*y
c5 advise her that the "enefit of controlling the e*ile*sy outweighs the ris& of
teratogenecity
d5 switch her to car"amaGa*ine
72% A *icture of a ni**le shoeing shiny a**earance and slight distortion of sha*e for one
year. #hat is the "est thing to do$
a5 Mammogram
"5 Aio*sy
c5 eassure the *atient
72% A college student comes to you with a history of fever and tonsillitis for two
wee&s. ,he too& am*icillin for one wee& "ut did not im*rove. !n examination she is
fe"rile/ loo&s ill and has several tender enlarged anterior cervical lym*h nodes. +hroat
examination reviled enlarged congested tonsils. ,he also has a maculo*a*ular rash on
the a"domen. +he most li&ely diagnosis isB
a5 ,u**urative tonsillitis
"5 Acute lym*ho"lastic anemia
c5 =nfectious mononucleosis
77% All the following is true a"out a *atient with dia"etic &etoacidosis exce*tB
a5 Patient usually *otassium de*leted
"5 aised #AC indicates an infection
..
79% A 1>%year%old "oy "rought to the E/ "ecause of edema and a"dominal
enlargement. @ou examine him and find *erior"ital edema/ ascitis/ and *rotienuria.
@ou admit him with the diagnosis of ne*hrotic syndrome and "egin thera*y. +hree
days later he "ecomes fe"rile and com*lains of a"dominal *ain. +he "est method for
diagnosing his condition is
a5 "lood culture
"5 urine culture
c5 a"dominal ta*
7<% A *atient comes to you with "ilateral &nee *ain and swelling/ he has stiffness in
the morning that involves the hands too. +he following investigations favor ,'E over
rheumatoid arthritis exce*tB
a5 negative rheumatoid factor
"5 shoulder involvement
c5 a"sence of thic&ened synovial mem"ranes
d5 su"cutaneous nodules
9>% A .2%year%old male *atient came to you "ecause of a *ainless swelling of his left
testisJ he gives history of mild trauma few days earlier. +he swelling is cystic/
involving the testis only and is not transillumina"le. +he diagnosis is
a5 varicocele
"5 tumor
c5 s*ermat*cele
d5 hydrocle
91% A *ara = *atient who delivered 2 months ago develo*ed *ost*artum hemorrhage.
,he tried to "reast%feed "ut was unsuccessful. ,he is now com*laining that her
*eriods have not resumed since the delivery. !n doing hormonal assay the following
hormones will "e in its normal range
a5 E,)
"5 ')
c5 Prolactine
d5 +,)
e5 Progesterone
92% #hile inserting an =:C6 for a woman in your office she suddenly "ecomes
restless/ *ale and "egins to lose consciousness. +he main immediate action you
should do is
a5 emove the =:6
"5 =n?ect su"cutaneous e*ine*hrine
c5 Elevate her leg
d5 =n?ect e*ine*hrine intramuscularly
9-% A -> yo male *atient comes to your office for evaluation. )a says he is usually
with low self%esteem/ and tired/ and finds his ?o" "oring/ whereas in some days he is
.0
very excited and hy*eractive and somewhat eu*horic. +his man is most li&ely
suffering fromB
a5 Ai*olar disorder
"5 Ma?or de*ressive disorder
c5 ,chiGo*hrenia
d5 Cyclothymia
e5 Mania
9.% =ncreased incidence of heart"urn and :+= in *regnancy is due to which hormone$
a5 hCG
"5 estrogen
c5 *rogesterone
90% =ncreased level of al*ha%feto*rotein in the amniotic fluid occurs in association
with
a5 hydroce*halus
"5 microce*haly
c5 meningomyocele
92% the "est way to diagnose coarctation of the aorta is to
a5 measure AP in "oth arms and com*are
"5 measure AP in arms and legs and com*are
c5 chest x%ray
97% +he average caloric inta&e of a normal neonate "orn -7 w&s gestation is
a5 110 Kcal;&g;day
"5 10>
c5 1>>
d5 0>
99% A 22 yo woman come to your office with a com*lain of a constant tiredness "ut is
otherwise normal. @ou examine her and find a diffusely enlarged thyroid gland/ its
not hot or tender and no other masses are *resent. @ou send "lood for +,)/ which
comes "ac& elevated. @our next ste* isB
a5 "egin thyroxin re*lacement
"5 "io*sy the gland
c5 re4uest a nuclear scan
9<% +he "est *ro*hylaxis for a Canadian visiting an area endemic in cholera would "e
B
a5 not eat anything unless *ro*erly coc&ed
"5 "oil water "efore drin&ing
c5 use oral tetracycline
d5 wash vegeta"le thoroughly

.2
1<<7 Canadian +est
1. ,omeone *resented to you with hy*o&alemia you would ex*ect all of the
following to occur exce*tB
% Muscular wea&ness
% Paralytic ilieus
% +%wave inversion/ short ,+/ :%wave
%6iarrhea Q
2. A "a"y "itten "y his *it dog/ what to do$
%Confine the dog and watch for ra"ies signs and sym*toms Q
,acrifice the dog with "rain examination for negric "odies
(accinate against ra"ies
a"ies immunoglo"ulin
-. 'ive attenuated vaccines are the following exce*tB
+A
@ellow fever
Polio
MM
6i*htheria Q
.. =n *eo*le *reviously ade4uately immuniGed against tetanus. )ow often "ooster is
given
Every 1> years Q
Every five years
0. outine immuniGation enclosed all of the following exce*tB
6P+%P
)i"
MM
!P(
Pneumovac Q

2. ). =nfluenGa meningitis contacts. )ow do you mange themB
ifam*icin orally for 2 days Q
ifma*icin to all households for . days
7. Pregnant woman 9%w& gestation with un&nown ru"ella titer came in contact with a
"oy with sus*ected ru"ella infection. #hat do you do$
Examine the "a"y for ru"ella A"
.7
6o anti"ody titer in the *regnant lady
Q
Give ru"ella =gG
Give ru"ella vaccine
9. Earmer came in the E with a nail in his foot and a tem*erature of -9.2 what to do
next$
%1 Give =v anti"iotics
%2 +etanus toxoid and antitetanus =g8s Q
<. outine test in the first *renatal visit includesB
% h test and anti"ody test Q
1>. @oung lady with menorrhagia/ commonest cause isB
% 6efiant leuteal *hase Q
%- ,hort secretory *hase
%. 6ysfunctional uterine "leeding
11. #hat is the most "eneficial treatment in the *revention of osteo*orosis in the
*ostmeno*ausal woman$
% Estrogen re*lacement Q
%0 Calcium
%2 (it 6
%7 ,to* smo&ing
%9 ,to* steroids
12. Postmeno*ausal woman had anterior myocardial infarction. ,he is a smo&er and
doesn8t want to sto* smo&ing. #hat are you going to advise her to do$
% ,to* estrogen re*lacement thera*y and start exercise Q
1-. Post%o* *t continued to "leed des*ite 1> units of *ac&ed AC transfusion. #hat is
the cause of the continued "leeding$
% 6ilutional throm"ocyto*enia Q
%< )y*ercalcemia
%1> )y*er&alemia
1.. Pt wit 6=C/ what is the most sensitive test
%11 AP++
%12 P+
%1- =H
%1. E6P Q
10. Pt with rheumatoid arthritis on A,A develo*ed ,'E. #hat is the treatment
%10 ,to* A,A
%12 =ncreased A,A
%17 ,tart high dose steroids
.9
%19 ,tart low dose steroids Q
12. Pt develo*ed excessive menstrual "leeding and *ur*uric rash in the leg. All the
following "lood tests will "e ex*ected exce*tB
%1< 'ow hemoglo"in
%2> )igh *latelets Q
%21 Prolonged "leeding time
17. #hat is the "est CH, rela*se *ro*hylaxis "rain tumor
% Arain chemothera*y
19. Pt with cervical and groin lym*hadeno*athy/ comes with wt loss and on
examination has s*leenomegaly and leu&ocytosis. #hat is the confirmatory test$
A%
"one marrow as*iration.
A% lym*h node "io*sy.

3C'' focal or diffused lym*hocytic infiltration/ 'ym*homa usually in the young and "one lym*h node "io*sy
is indicated/ "ut mostly AM as* is done5

1<. Pt with "ilateral hilar lym*hadeno*athy with micronodular infiltrate. #hat is
the diagnosis$
a% trans%"ronchial "ronchogenic carcinoma
"% sarcoidosis
c% *neumonia
d% scar tissue
2>. #hat is the treatment of "artholines a"scess$
a% marsu*ialiGation
"% incision and drainage
c% anti"iotic thera*y
d%
21. Pt with severe *erianal "luish mass *rotruding from the anus/ the *ain gets
worse after "owel motion or defecation. #hat is the diagnosis$
a% throm"osed external *iles
"% *rola*sed anal *iles
c% internal *iles
22. #hat is the commonest cause of *ruritis ani in the elderly$
a% fecal incontinence
"% laxatives
c% hemorrhoids
d% carcinoma of the rectum
2-. :nclear 4uestion on rectocele and cystocele
2.. A 2. years old lady with *ositive *regnancy test came with history of 2 days
vaginal "leeding. :ltrasound examination showed em*ty uterus with - cm
mass in the adnexa. ,ince then she did not have early morning nausea and her
vaginal examination showed a closed cervical os. #hat is the diagnosis$
.<
a% com*lete a"ortion and leuteal cyst
"% ecto*ic *regnancy
c% via"le *regnancy
d% inevita"le a"ortion
e% threatened a"ortion
20. #hich one of the following is an a"solute indication for cesarean section in
the "reach$
a% footling com*lete
"% fran& "reech and flexed head
c% *oor o"stetrician
d% first "reach of a twin
22. #hat is the definition of crude "irth rate$
a% the num"er of the total live "orn in a year over the num"er of the total
*o*ulation
"% the num"er of the total live "orne in a year over the total num"er of
children in res*ected age grou*
c% the total num"er of live "irth and still "irth in a year over the total
num"er of the *o*ulation.
d% the total num"er of live "irth over the num"er of the
females in the re*roductive years
27. #hat is the definition of *revalence$
a% the num"er of the existing cases 3old and new5 over the num"er of total
*o*ulation.
29. A 2- years old girl comes to consult you regarding her ris& to develo*
)untington8s chorea. )er father died with the disease/ so did her *aternal
grand father/ and her *aternal anti has ?ust develo*ed it. @ou till her that her
ris& isB a% 1>>D
"% 70D
c% 0>D
d% 20D
e% >D
2<. A figure drawn for a *edigree of family with a disease showing a vertical
transmission of the disease. #hat is the mode of inheritance$
a% autosomal dominant
"% autosomal recessive
c% x% lin&ed dominant
d% x% lin&ed recessive
e% *olygenic inheritance
->. A -> yo lady with 2 wee&s amenorrhea/ *resented with excessive early morning
vomiting/ and a"dominal distension. :ltrasound of *elvis showed gra*e li&e
masses in the uterus. All of the following can cause it exce*tB
a% hy*othyroidism and *rior"ital mexodema
"% multi*le gestations
0>
c% "ilateral ovarian cysts
d% *reclam*sia in the first trimester
-1. All of the following are ris& factors of *reeclam*sia exce*tB
a% Multi*arity
"% 6m
c% null Para
d% history of *revious *reeclam*sia
e% essential hy*ertension
-2. A *regnant lady 2> wee&s gestation. )er *u*ic%fundus height is 20cm. All of the
following is true exce*tB
a% normal *regnancy
"% wrong estimation of gestational age
c% twins *regnancy
d% *olyhydramnios
e% Gestational tro*ho"lastic neo*lasia
--. -0 years old comes to the clinic with severe a"dominal *ain/ fever/ and *ainful
intercourse. !n examination she had multi*le white lesions in her cervix. #hat is
the diagnosis$
a% chlamydia cervecitis
-.. Question on P=CA
-0. Pt with internal carotid artery stinosis/ is li&ely to suffer from$
a% Amarusis fugax
-2. +he most im*ortant indicator of a tumor *rognosis isB
a% grade of the tumor and histology
"% extension to irregular lym*h nodes
c% clinical stage of the tumor
-7. +he "est *lace for a *a* smear sam*le isB
a% endo and ecto cervix
"% endocervix
c% ectocervix
d%endocervix and *osterior vaginal wall
e% endocervix and *osterior vaginal wall
-9. 'eft illeofemoral throm"osis 1> days *ost o*eratively/ with left extensive swelling
and *ain from the groin to the foot. 3=.e. 6(+ extending to the illeofemoral valve5
a% *hlegmasia cerula dolans
"% *hlegmasia al"a dolans
01
-<. Pt 1> days *ost o*/ with shortness of "reath and chest *ain. )is tem*erature is
-7.7C.
a% C1 most li&ely will "e normal
"% he has *ulmonary em"olism
c% (;Q scan is highly s*ecific
.>. 2< years old lady with left an&le edema. Com*lains of a su*erficial ulcer at her
medial malleolus/ with a surrounding scar. #hat is the underlying *athology$
a% venous valvular insufficiency
"% venous insufficiency
c% incom*etent *erforating veins
.1. Pregnant lady with a &nown fi"roid came with a"dominal *ain. #hat is the
diagnosis$
a% red degeneration
.2. #hat is the treatment of decom*ression
a% recom*ression
.-. - yrs old child with cough ins*iratory stridor/ -%day history of fever and now is in
res*iratory distress. +he chest x%ray shows thum" sign. #hat is the diagnosis$
a% acute e*iglottitis
"% adenoids
c% di*htheria crou*
d% tracheo laryngo "ronchitis
... Picture of a child who ingested a cleaning agent in the &itchen. )e has an ulcer in
the mouth and the li*. #hat is your management$
a% eso*hagosco*y
"% "arium study
.0. 12%year%old "oy with history of weight loss/ *assing red "loody diarrhea/
arthralgia. Colonosco*y and sigmoidosco*y were normal. #hat is your next line
of management$
a% u**er G= studies
.2. a child with fat mala"sor*tion/ a"dominal "loating/ foul smelling stool/ and loss of
weight/ and easily "ruising. #hat is wrong with the child$
a% gluten entero*athy
"% G= lym*homa
c% G= tu"erculosis
d% re*eated e*isodes of *ancereatitis
3he did not mention cystic fi"rosis5
.7. dia"etic *t with dia"etic &etoacidosis. All of the following occur exce*tB
a% =ncreased serum amylase
02
"% a"dominal *ain
c% decreased serum *otassium
d% increased osmolality
e% dehydration
.9. a ty*e = dia"etic mother gives "irth to a child and she as&s you for instructions.
@ou will advice her of all of the following exce*tB
a% hold the insulin until the "a"y starts feeding.
.<. A .%year%old child with a *osterior mediastinal tumor. +he commonest differential
isB
a% neuro"lastoma
"% thymoma
c% germ cell tumor
d% lym*homa
0>. A *atient had a trauma to his testicles in the *ast. How he com*lains of a *ainless
testicular mass. !n examination you get a"ove it and can se*arate it from the
e*idydimis/ and it does not transiluminate. #hat is the diagnosis$
a% seminoma
"% s*ermatocele
d% chronic e*idedimitis
01. +he "est *lace for ta&ing a smear for gonorrhea isB
a% cervix
02. A *t *resented with a heavy/ dragging feeling in his testicle that ma&es him
uncomforta"le. And he has a *ainless swelling in his right testicle. #hat is the
finding that goes with a hydrocele diagnosis$
a% transillumination is *ositive
"%
0-. At which stage of sexual excitement/ according to +oron criteria/ the woman
develo*s clitoral erection/ increased secretion/ and ni**le erection$
a% early excitement
"% late excitement
c% resolution
d% latent *hase
0.. 'ady with history of difficulty carrying sho**ing "ags/ wea& thum" a"duction/ and
thinar eminence wasting. #hat is the diagnosis$
a% Car*al tunnel syndrome
"%
00. Pt with radial verve in?ury at the radial head level. #hat is the o"served lesion$
0-
a% wrist dro*
02. Pt with dro* foot. )e gives a history of falling of his tight side and lacerating the
lateral *art of his &nee. #hat is the cause of his dro* foot$
a% common *eroneal nerve *alsy.
"% '0%,1 dis& *rotrusion
c% C(A
07. Question on sexual desire a"normalitiesB
a% does not have sexual fantasies
"% no genital activation for 9 wee&s
c% nervous a"out it
d% try to avoid *arents contact
09. =n transexualism all of the following is true exce*tB
a% they are feminism
"% childhood history of cross dressing
c% they are heterosexual men
d% many have a normal marriage life
c% *artner is coo*erative
3transexualismB strange *ersistent desire and insistence to "e of the
o**osite sex. 6ives history of cross dressing/ cross ha"its/ and change of game
rules5
0<. All of the following drugs cause de*ression exce*t
a% cyclos*orin
"% al*ha methyl do*a
c% clonidine
d% cemitidine
e% *ro*ranolol
2>. Psychothera*y is useful in all of the following exce*tB
a% organic mental disorder
"%
21. Pt "rought to you mute and inaccessi"le for examination. #hat is the diagnosis$
a%
catatonic schiGo*hrenia
22. Pt came to you in E with catatonic sym*toms and slowness of thin&ing/
decreasing levels of consciousness/ a*athy/ with few sym*toms of a*athy. #hat is
the diagnosis$
a% A=6, com*lex
2-. @oung -.yo *t with decreased motor activity/ recent history of short%term memory
loss/ decreasing levels of consciousness/ ataxia/ and wea&ness. #hat test will you
0.
do for him$
a% )=( test
"% C+ "rain
c% EEG
2.. 0< years old *t came with a"ru*t onset of focal neurological deficit/ associated
with *atchy cognitive manifestation. #hat is the diagnosis$
a% multi*le infarct dementia
"% *seudo dementia
c% AlGheimer8s disease
d% A=6, dementia com*lex
e% multi*le sclerosis
20. )ow can you differentiate "etween multi infarct dementia and AlGheimer8s
disease$
a% a"ru*t onset
22. A -0 year old male found in the street RwonderingS. All of the following are
*ossi"le etiologies exce*tB
a% AlGheimer8s disease
"% schiGo*hrenia
c% fugue
d%
27. Eemale *t with dou"le swallowing and gargley sensation in the mouth when she
eats. #hat is the diagnosis$
a% crico*harengeal diverticulum
"% hiatus hernia
c% achalasia
d% eso*hageal carcinoma
e%
29. Picture of a girl with a midline nec& swelling that goes u* when she *rotrudes her
tongue.
a% thyroglossal cyst
"% dermoid cyst
c% "ranchial cyst
2<. +he following is routine investigation for an elderly lady with urinary
incontinence/ Exce*tB
a% direct visualiGation of incontinence when the *t is in full "ladder while
coughing
"% urine analysis
c% cystourethrogram
d% urinary diary
7>. All of the following cause dysuria E1CEP+B
a% *yelone*hritis
00
"%cystitis
c% urethritis
d% vaginitis
e% +)E answer is not availa"le
71. +he definitive test for chronic *ancreatitis isB
a% ECP
"% C+
c% M=
d% :;,
e% serum amylase
72. .- yo male with h;o re*eated right u**er 4uadrant tenderness. How he *resented
with mass in the same *lace "ut there is no *ain or itching. +he mass descends
during ins*iration. #hat is the diagnosis$
a% mucocele of gall "ladder
"% he*atoma
c% cholecystitis
d% carcinoma of the gall"ladder
7-. Pt came with *ruritis and *rogressive ?aundice/ with raised A'K *hos/ and
"iliru"in. #hat is the "est test you will do$
a% :;,
"% ECP
c% P+C
7.. )ow will you manage a *t with 4uadrice*s femoris hematoma$
a% drainage
"% heat ex*ression
c% cold
70. P=C+:E of a child with :MA='=CA' hernia. #hat is the management$
a% reassure the mother and re%examine in 12 months
72. A young child was "rought with a varus deformity 3clu" foot5 of his foot. #hat is
the management$
a% *arents need to do *assive flexion movement
"% cast
c% nothing needs to "e done as it will correct it self
77. A lady 1> days *ost *artum develo*s a swelling and tenderness of the right calf
muscle. ,he has "een "reast%feeding and now she has a low%grade fever. +his *t
can "e given all of the following exce*tB
a% early mo"iliGation
"% he*arin
c% comudin
02
79. #hich of the following drugs does not cross the *lacenta$
a% succinyl choline
"% H2!
c% mor*hine
d% demeral
e% diaGe*am
7<. A "reast%feeding lady with tender swelling in her right "reast. #hat is true
regarding her condition$
a% sta*h aureus is the *rimary organism
"% caused "y gram negative organism
c% needs incision and drainage
9>. All of the following are reasons for not *rescri"ing multivitamins to the *regnant
lady exce*tB
a% they are ex*ensive
91. =n the management of insect in the ear. All of the following can "e done exce*tB
a% *ut water in the ear
"% removal "y force*s
c% consult EH+ surgeon
d% use oil to &ill the insect
e% otosco*y
92. Pt develo*s recent attac&s of tinnitus/ vertigo/ hearing loss/ and nystagmus. !n
examination he has constant horiGontal nystagmus. #hat is the diagnosis$
a% meiners disease
"% acoustic neuroma
c% "enign *ositional vertigo
d% la"irinthitis
9-. Pt came with chronic "ronchitis/ given erythromycin for 1> days. How he is
com*laining of sever earache. #hat is the diagnosis$
a% transient auditory im*airment
"% mastoiditis
c% ottitis media
9.. Commonest cause of *seudo%mem"ranous colitis isB
a% clostridium toxin ty*e A
90. =n necrotiGing enterocolitis all of the following features *resent exce*tB
a% *eritoneal calcification
"% *neumatosis intestinalis
c% gas in *ortal vein
07
d% *neum*eritonium
92. Pt suffers from recurrent swelling of li*s and eyelids/ generaliGed erythema and
itching/ and some mem"ers of the family have a similar condition. #hat is the
diagnosis$
a% C1 esterase inhi"itor deficiency
97. +he relationshi* "etween as"estos and smo&ing isB
a% synergistic
"% additive
c% cumulative
d% antagonist
e% neutral
99. All of the following cause lung cancer exce*tB
a% lead
"% mustard gas
c% arsenic
d% coo& oven emission
9<. =n "attery factories the *oisoning is *rimarilyB
a% lead
"% mercury
c% car"on monoxide
<>. Picture of a young girl with a non%scarring allo*ecia at multi*le areas. Commonest
cause isB
a% allo*ecia areata
"% tenia ca*itus
c% chemothera*y
<1. 6uring insertion of =:C6/ the lady develo*ed hy*otension. #hat are you going to
do immediately$
a% elevate the legs
"% give her ringer8s lactate
c% *ut her in telendlen"urg *osition
d% remove the device immediately
<2. =:C6 thread seen dangling outside the cervix during examination of a *regnant
lady. =f it is removed/ what is going to ha**en$
a% normal *regnancy
32>D normal *regnancy/ 20D a"ort during removal/ 10D s*ontaneous
a"ortion5
<-. Pt came to the E with T of femur/ develo*ed sudden hy*otensi*on. #hat is the
most li&ely cause$
09
a% "lood loss
<.. (era*amil 0 mg given =( for treatment of ,(+. +he *t develo*s hy*otension/ what
will you do$
a% Cardioversion
" I "eta "loc&er iv
c% digoxin iv
d% iv fluids
3if cardiogenic shoc& or heart failure cardioversion/ 1
st
drug of choice adenosine/ es*ecially with #P#/ if
not vera*amil/ meto*rolol/ then digoxin5
<0. +inen crwu8s x
a% local ter"inafine
<2. Picture of a hand with a nevus at the ?unction of the *alms. =t has irregular "order/
shar*ly demarcated macule/ and dar& "rown in color. #hat is the diagnosis$
a% ?unctional nevus
"% melanoma
<7. Picture of s&in lesion on the lower li*.
a% s4uamous cell carcinoma
"% "asal cell carcinoma
<9. 6iaGe*am has a cross tolerance withB
a% alcohol
<<. Pt too& an over dose of a *sychostimulant. #hat is the "est way to decrease
com*lication when he goes home$
a% daily follow u* in a su**ort grou*
"% "enGodiaGe*ines
1>>. An alcoholic o came to the emergency de*artment with sym*toms of delirium
tremens. @ou would ex*ect all of the following to occur exce*tB
a% 6elusions
"% tremor
c% visual hallucinations
d% *sychomotor activity
e% fever
1>1. :nclear 4uestion on organic *ersonality syndrome
3frontal lo"e syndrome
% irrita"ility or a*athy 5
% eu*horia 5
% tal&ative and hy*eractive 5
% disinherited 5
0<
% *sychothera*y is least effective treatment 5

1>2. +he defense mechanism in *ho"ia isB
a% dis*lacement
"% controlling
c% dissociation
1>-. =n conversion disorder all of the following is true exce*tB
a% *ro?ection
"% intellectualiGation
c% 'a"elle indifference
1>.. =n contact dermatitis all of the following is true exce*t
a% local steroids are not useful
"% systemic steroids could "e used
c% mostly associated with *lant ex*osure
d% s&in test is useful
1>0. All of the following can cause inverted uterus exce*tB
a% =( oxytocin
"% atonic uterus
c% *ressure on s soft fundus
d% forceful *ull on the Cord
e% grand multi*ara
1>2. #ho is res*onsi"le for the ultimate health care and 4uality in the hos*itals in
Canada$
a% "oard of directors
"% staff
c% chief staff
d% CE!
3+)E )!,P=+A' care is as followsB Aoard of directors Uadministrative teamV res*onsi"le for
administrative services e.g. record admission
education
nursing and medical affairs
diagnostic and thera*eutic services
1>7. A child "rought "y his *arents/ you sus*ect a"use. #hat is the first thing you
would do$
a% contact child *rotection agency
1>9. Pt with *lasmodium vivax malaria/ treatedJ now he has recurrent sym*toms. #hat
is the case$
A% no eradication from the liver

2>
1><. =n *t with "icornoate uterusB
a% most of the cases have normal *regnancy
11>. +he role of the royal college of *hysicians in Canada isB
a% care of the *ost graduate s*ecialty
111. A 00 yo lady with whitish discoloration of the vulva and itching. #hat is the next
line of management$
a% eccesional "io*sy
"%
112. =n a child with urticaria you will tell the *arents that most of the cases have an
un&nown etiology/ and
a% case will resolve
11-. +he "est screaming test for *rimary hy*eraldosteronism isB
a% serum renin activity level
11.. Pt was a"using alcohol over the wee&end and was on A,A/ develo*ed G=
"leeding. Gastrosco*y showed multi*le areas of gastric erosions. All of the
following can "e done exce*tB
a% gastrectomy
"% HG tu"e
c% vaso*ressin
d% )2 "loc&ers
110. :nclear 4uestion on A,A thera*y
a% "etter tolerated in children
"% serum thera*eutic level reached in 2>%-> mins
c% an evening dose can "e esca*ed
3%usesB 1% *revention of M=;+=A 2% x of throm"osis -% heumatoid diseases as H,A=6
% C;=B 1% asthma O nasal sensitivity 2% P:6 -% *reg .% children
% ,;EB 1% G=B H;(/ P:6/ dys*e*sia 2% earB tennitus/ vertigo/ hearing loss
-% "loodB anemia/ leu&o*enia/ +CP/ *ur*ura .% nasal *oly*s
112. #hat is the commonest cause of maternal mortality in Canada$
a% Advanced maternal age
"% *ost *artum hemorrhage
c% *reeclma*sia
d% multi*le *regnancies
117. Question on strea&s on a C1 showing Rtram trac&ingS and *arallel lines
radiating from the hilum/ and dilated "ronchi with thic&ened walls. #hat is
the diagnosis$
a% Aroncheictasis
21
119. Cough with localiGed wheeGing and clu""ing in a *t with C!P6
a% "roncheictasis
11<. a female *t with AHA *ositive/ an&le edema/ *rotein in urine and arthralgia.
#hat is the diagnosis$
a% ,'E
"% A
12>. AHA *redict uveitis develo*ment inB
a% ?uvenile rheumatoid arthritis
121. #hat is the management of a full term *regnant lady with active her*es lesion
in the vagina$
a% elective cesarean section
122. All of the following are *redis*osing factors in the intrauterine growth
retardation exce*tB
a% low maternal weight gain
"% history of *revious =:G
12-. All of the following can cause *olyhydramios exce*tB
a% renal agenesis
"% 6M
c% multi*le gestations
12.. Question on gastroenteritis in school children. #hat is the organism$
a% sta*haureus
120. Question on the oc&y Mountains/ and diarrhea.
a% giardia lam"lia
122. Question on a study that has Rno control casesS
127. Advantage of the ex*erimental study isB
a% can decide who will "e ex*osed to the ris& factors and who will "e not
129. All of the following can "e the treatment of condylomata acuminata in
*regnancy exce*tB
a% *odo*hylin
12<. agtra attac&
a% sulfate
1->. A study that identifies ris& factors *rior to start isB
a% cohort study
1-1. #hich is false in health care in CanadaB
a% more hos*itals needed
22
1-2. #hat is the *revalence of a disease$
a% the num"er of the existing cases of a disease divided "y the mid year
*o*ulation
1--. +he most common cause of lower res*iratory tract infection in infants isB
a% res*iratory sincytial virus
1-.. Pre%renal aGotemia
a% G.urine N 1.>1.
1-0. +he elderly community in CanadaB
a% they *refer to stay alone at home and "e visited "y mem"er of their family
1-2. Pt "rought to the E6 with decreased level of consciousness/ *in*oint *u*il
and shallow "reathing. #hat is the diagnosis$
a% heroin intoxication
1-7. ECG showing P% M >.2/ Q, wide N >.12 fast rhythm. #hat is your
management$
a% Carotid massage
"% adenosine
1-9. Child with headache and a"dominal *ain/ nausea and vomiting the slee*s.
#hat is the diagnosis$
a% childhood migraine
1-<. Pt came with a history of trauma/ and then after a lucid interval he develo*ed
coma. #hat is the cause$
a% e*idural hematoma
"% ex*anding intracranial hematoma
1.>. Huchal headache/ rigidity/ and unilateral *u*illary dilatation. #hat is the
diagnosis$
a% su"arachnoid hemorrhage
1.1. -
rd
nerve *alsy with *reserved *u*illary light reflex. #hat is the diagnosis$ a%
6M
"% com*ressing "rain lesion
1.2. Pt with right ulnar and radial nerve *alsy and hornorCs syndrome. #hat is the
cause$
a% "ronchogenic carcinoma
"% cervical ri"
c% thoracic outlet syndrome
d% hand%shoulder syndrome
1.-. =n anemia of chronic disease all of the following is true exce*tB
a% folate deficiency
2-
1... +he most sensitive and s*ecific test for *rimary sy*hilis isB
a% E+A%AA,
"% +P=
c% (6'
d% @,'M
1.0. Pt with ottitis media develo*s coma and convulsions. #hat is the first to do$
a% lum"ar *uncture
1.2. =n a child with hy*ertonic dehydration/ what is the commonest com*lication
that might arise due to treatment$
a% hy*erthermia with convulsions
"% cere"ral edema
c% arrhythmias
1.7. #hich of the following vitamins is most toxic if given to an infant in the
following doses$
a% (it K 1 mg =M for 1 wee&
1.9. (it & deficiency in new"orn you will findB
a% raised P+
1.<. Pt with a *icture of talangectasia/ and eynolds *henomena. 1%ray of hand
show soft tissue calcification. #hat is the diagnosis$
a% scleroderma
10>. Proximal muscle wea&ness and grottons *a*ules. #hat is the investigation of
choice$
a% muscle "io*sy
"% EMG
101. =n a child with hydroce*halus/ &aryoty*ing will most li&ely showB
a% eu*loidy
102. =n *oly myositis the investigation of choice is
a% EMG
"% ,K
10-. A *t with wea&ness and *ain in the left arm now develo*ed "ilateral lower
lim" *ain. #hat is the diagnosis$
a% cervical s*ondylosis
10.. =n *rola*sed disc at '.%0 you will ex*ect the *t toB
a% loose &nee reflex
"% loose hamstring reflex
c% loose an&le reflex
2.
100. C1 showing tension *eumothorax. ,e*aration of visceral and *arietal *leura
3density5/ hy*er lucent small dilated lung/ with lac& of *eri*heral lung
mar&ings. #hat is the management$
a% needle as*iration
"% chest tu"e drainage
102. C1 showing hy*erlucent left lung with mediastinal shift. +he child is
coughing and wheeGing. #hat is the diagnosis$
a% EA left lung
"% *neumonia
c% left hy*erlucent lung syndrome
107. #hich of the following is most suggestive of thyroid malignancy$
a% ra*idly enlarging node within one month
109. #hich of the following will mostly affect the future *lanning of Canadian
health *rogram.
a% age
10<. A child who is very meticulous/ very clean/ very well general a**earance/ and
gets very anxious in the *eriods *rior to the exams. #hat is wrong with him$
a% over anxious *ersonality
12>. =n *ost *artum *sychosis/ all of the following is true exce*tB
a% can cause residual effect
121. +he "est antide*ressant in coronary artery disease with least antichlenergic
side effects isB
a% Eluxetine
122. +he "est treatment of shigellosis isB
a% +MP;,M1
"% se*trin
c% am*icillin
12-. Aest indicator of 6M isB
a% decrease in wt des*ite increase in a**etite
12.. All of the following can cause laryngeal *oly*s exce*tB
a% *resent medications
"% )P( 2/11
c% hormone thera*y
d% congenital
e% vocal *ursuit
20
120. A child was "orn and found to "e flo**y. #hat is the most li&ely cause$
a% triosomy 21
122. A *icture of a child with downs syndrome. #hat are you going to do for him$
a% &aryoty*ing
127. A cou*le had a new"orn. +he wife has a mother who is schiGo*hrenic. #hat
are you going to advice them$
a% can have childrenJ they will have slight increase in the ris& of disease in
the offs*ring.
129. A lady/ who is e*ile*tic and controlled on *henytoin/ gets *regnant. #hat is
your advice to her$
a% "enefit of use out way its teratogenecity
12<. #hat is true in tetracycline toxicity$
A% its dose and duration de*endent
"% no ris& of teeth discoloration in a single dose
17>. 19 year old with feature of hy*othyroidism
a% start '%thyroxin
"% do iodine u*ta&e studies
171. Aest screaming test for hy*othyroidism
a% +,)
172. Child develo*s acute e*isode of crying lifts his leg u*ward. 1%ray shows
reversed E%sign. #hat is the "est managementB
a% Aarium enema
17-. 0 year old "lac& male/ com*laining of 0 day history of "ilateral non%*urulent
con?unctivitis/ red fissured li*s/ straw"erry tongue/ and rash. #hat is the
diagnosis$
a% Kawasa&i disease
17.. A man had a fight at wor&. )e was "rought to the hos*ital com*laining of
"ac& *ain. !n examination no *athology was found. )e is de*ressed and
o"sessed with it and insisting on having a re*ort. #hat is the diagnosis
a% malingering
"% hy*ochondriasis
c% somatoform *ain disorder
170. Question on *lace"o
172. =n malingering you will ex*ect the *t to have
a% external gain
22
177. A lady feeling dissociated and confused from the environment and cant find
the wards she wants to say. #here is the lesion$
a% left frontal
"% left tem*oral
c% right tem*oral
d% left *arietal
179. Pt who is charming/ mani*ulative and does not learn from his *revious
ex*eriences/ and lac&s remorse. #hat ty*e of *ersonality disorder is it$
a% antisocial *ersonality
"% histrionic *ersonality
17<. =n acute iritis/ you will find all of the following exce*tB
a% discharge
"% *hoto*ho"ia
c% ocular *ain
d% decreased visual acuity
e% *artially constricted *u*il
19>. =n *t with hy*ochondriasis
a% they "elieve of having a chronic disease des*ite good reassurance
191. A mother "rings her "a"y to you in the clinic. )e is not sensitive to the
mother8s feelings/ and having a *ro"lem with learning s*eech. #hat is the
diagnosis$
a% Autism
"% learning disorder
192. =n avoidance *ersonality disorder/ you will findB
a% they are sensitive to criticism "y others

19-. #hat is true regarding attention deficit hy*er activity disorderB
a% most of the cases *ersist to adulthood
"% they are antisocial
c% have a criminal tendency
19.. Pt comes with chills/ fever/ and ?aundice. #hat is the diagnosis$
a% ascending cholangitis
190. #hich of the following is associated with an unex*ected/ non%traumatic
death in otherwise clinically sta"le *t within one hour of sym*tom onset$
a% e?ection fraction M 2>D
"% multi*le ventricular arrhythmias on 2. hour halter monitor
c% family history of sudden death
d% atrial fi"rillation with *revious anterior myocardial infarction
27
e% severe narrowing of more than one coronary artery
192. #hich of the following is the "est in decreasing M(A
a% road side alcohol "reath test
"% seat "uilt
197. All of the following have a higher ris& of com*leted and successful suicidal
attem*t exce*tB
a% female sex
"% de*ression
c% malignancy
199. All of the following drugs can cause mania exce*tB
a% car"amaGe*ine
"% '%do*a
c% steroids
d% amitri*taline
19<. #hat is the treatment of choice for cyclothymia
a% 'ithium
1<>. Most im*ortant dietary modification in middle age grou* is$
a% total calories content
"% fi"er diet
c% C)! content
d% *rotein
1<1. =n multi*le sclerosis all of the following occur exce*tB
a% an osmia
"% tremor
c% di*lo*ia
d% nystagmus
e% scotoma
1<2. =n acute alcohol intoxication all of the following occur exce*tB
a% tongue tremor
"% "radycardia
c%
1<-. =n Aulimia nervosa all of the following occur exce*t
a% meta"olic acidosis
"% dental caries
c% G= reflux
d% *arotid enlargement
29
1<.. (isual hallucinations most li&ely associated withB
a% delirium
"% organic "rain disease
1<0. !f the following ion a man with P(6/ which of the following is a sign of an
im*ending gangrene$
a% rest *ain
1<2. A *t alcoholic with liver disease develo*s "ilateral 2
th
nerve *alsy/
nystagmus/ and ataxia. #hat is the diagnosis$
a% werni&e8s ence*halo*athy
"% he*atic ence*halo*athy
c% &orsa&off ence*halo*athy
1<7. Pt with history of "rochogenic carcinoma develo*s cough/ high AP/
vomiting/ headache worse in the morning. #hat is the diagnosis$
a% "rain metastasis
1<9. All of the following is true regarding normal grief exce*t
a% it can last u* to 12 months
1<<. =n dysthymia
a% ma?or de*ressive e*isodes doesn8t occur
2>>. =n rhesus incom*ati"ility
a% s*ectrmatometry
"% amniocentesis
2>1. Pt develo*ed meningitis/ and then found to have low sodium. #hat is the
reason for it$
a% syndrome of ina**ro*riate A6) secretion
2>2. Pt *resented with *ainful red eye with a stic&y secretion. #hat are you going
to do$
a% start to*ical anti"iotics and follow u* in the clinic
"% do tonometry
2>-. (aginal "leeding in a *ost *artum lady within first hour. #hat is the *ossi"le
cause$
a% retained *lacental fragment
31st 2. hour "leed/ mostly uterine atony. 'ate *ost*artum "leed mostly
uterine involution5
2>.. @oung *t with &nee effusion and history of "ac& *ain. #hat is the most
a**ro*riate investigation$
a% sacroiliac ?oint x%ray
2<
2>0. )ow does a *t with retinal detachment descri"e it$
a% shadow in front of the eye
2>2. #hat is the initial management of frost"ite$
a% re%worm in .- degree water

2>7. Question on a "a"y with feeding *ro"lem
2>9. #hat is the management of a &id "rought com*laining of a localiGed *ain to
the um"ilicus$
a% reassurance
2><. A "oy fell down from a tree. )e started to have a"dominal *ain and
develo*ed hematuria. #hat is the most a**ro*riate management$
a% =(P
"% renal scan
c% retrograde uro*ath
d% surgical ex*loration
21>. Question on =:G assessment. Ay :,
211. =n wood factory the em*loyees are susce*ti"le to getB
a% sinus cancer
"% lung cancer
c% liver cancer
212. 0 yo child with stridor and res*iratory distress. 1%ray shows thum* sign.
#hat is the diagnosis$
a% acute e*iglottitis
"% acute asthma attac&
c% retro*harengeal a"scess
d% traheo%laryngo%"ronchitis
21-. Question on a *t who feels dissatisfied with "ladder em*tying. $ !ver flow
incontinence
21.. 6ifference "etween mania and schiGo*hreniaB
a% thought "roadcasting
210. Pt with dementia. #hat of the following is a reversi"le cause of dementia$
a% *ernicious anemia
212. Pseudo dementia
a% de*ression
217. Gra*h of *oint source e*idemic with secondary cases
219. Picture of vulva with white rash R mostly with surrounding red rashS
a% her*es
7>
"% molluscum
c% sy*hilis
d% +A
21<. All of the following is seen in "acterial vaginosis exce*tB
a% gray foul smelling discharge
"% fishy odor when K!) is added to the discharge
c% causes intense cervisistis
d% *aucity of lacto"acilli
22>. Question on *lacenta *revia
a% "ed rest in hos*ital
221. A"ru*tio *lacenta
a% *ainful vaginal "leeding
222. #hich one of the following suggests endometriosis on examination$
a% *ainful
22-. Picture of du*ytrun8s contracture
22.. Penetrating wound to a finger of a child with severe tenderness and swelling
to the finger/ and *ainful extension. #hat is the diagnosis$
a% se*tic tenosinovitis
"% *ul* s*ace infection
220. Pt with diaGe*am withdrawal/ which of the following you can8t give$
a% "us*irone
"% diaGo*oxide
c% loraGe*am
222. )ow can you avoid "utulinism in food$
a% nitric *reservatives
"% antacids
c% "elow freeGing tem*
d% vacuum food
227. Post gastrectomy *t on oxygen. )is !2 Hormal/ C!2 raised. #hat is the
cause$
a% res*iratory disease
"%com*ensated res*iratory al&alosis
c%meta"olic acidosis
229. Hew"orn "a"y with cyanosis/ otherwise is healthy/ you would sus*ectB
a% trans*osition of the great arteries
"% (,6
c% P6A
71
d% hy*o"lastic left heart
22<. Hew "orn with congestive heart failure without cyanosis. #hat is the li&ely
cause$
a%(,6
"%P6A
2->. )ow would you "e a"le to diagnose a new"orn with deafness$
a% evo&ed cere"ral *otential
2-1. +he organiGation that is res*onsi"le for the control of the drugs in Canada isB
a% association of *harmaceuticals in Canada
2-2. 'ady with "ilateral lower a"dominal 4uadrant *ain and tenderness after the
end of the cycle "y 2%- days. #hat is the diagnosis$
a% sal*ingitis
2--. ,odium cromoglycate hel*s in asthma "yB
a% sta"iliGing the mast cells
2-.. Question on hay fever
2-0. #hat is the management of sic& "uilding syndrome$
a% *rom*t ventilation of the "uilding
2-2. =n acute *ost stre*tococcal glomerulone*hritis/ all of the following is
consistent with it exce*tB
a% no evidence of recent :+=
"% increase in AP
c% increase in A:H
d% decrease in C-/ C.
2-7. Question on C1 esterase inhi"itor deficiency
2-9. All true in )untington8s disease exce*tB
a% onset in mid sixties
2-<. Commonest cause of intestinal o"struction in a male N 2> year old isB
a% *ost surgical adhesion
2.>. A child swallowed a screw 2.0 cm in lengthJ x%ray shows it in the right u**er
4uadrant region. #hat is the "est management$
a% leave it and re*eat the x%ray in few days
2.1. Chlamydia is aB
a% "acteria
"% virus
72
c% *arasite
2.2. !vulation
a% ') urine measurement
2.-. A girl with a small tissue "ehind her ni**le. #hat is the management$
a% reassurance
2... All of the following is true regarding fi"rocystic disease of the "reast/
exce*tB a% needs hormonal treatment
2.0. 2>%year%old female with a *ainless hematuria/ with clots/ fre4uency/
nocturia/ and dysuria. #hat is the diagnosis$
a% Ca of "ladder
2.2. =n *remature ru*ture of the mem"ranes. #hat is the *ossi"le com*lication$
a% se*ticemia
"% *neumonitis
2.7. Aoy with interstitial *neumonia unres*onsive to *enicillin. #hat is the
reason$
a% myco*lasma *neumonia
2.9. #hat is the im*ortance of randomiGation in a controlled trial study$
a% to eliminate the &nown confounding factor "etween the control and the
study grou*
2.<. Pt fell on out stretched hand/ x%ray normal/ "ut he continues to have severe
*ain with limitation of hand function. #hat is the management$
a% a**ly cast and re*eat x%ray after 2 wee&s for sca*hoid view
20>. A 0%year%old "oy descri"ed to "e healthy/ the family moved to Canada and
he started develo*ing enuresis. #hat is the "est treatment for him$
a% "ehavioral thera*y
"% imi*ramine
c% family thera*y
201. A female .1%wee& gestation comes to the clinic. )as no signs of la"or. #hat
is the management$
a% see after one wee& and follow non stress test
202. A 12%year%old "oy with no sexual characteristics. !n examination/ his testes
are normal/ *enis siGe is normal/ and his *u"ic her is scanty. #hat is your
management$
a% reassurance
7-
20-. Gonorrhea treated with s*ectmicin/ "ut there is a recurrence of sym*toms.
#hat is the cause$
a% chlamydial infection/ give tetracycline
20.. Picture of nail *itting. #hat is the diagnosis$
a% *soriasis
200. ureteral stone with hydrone*hrosis. #hat is the management$
a% surgical extraction
"% ureteric lithotre*sy
202. A lady with sore shoulder/ headache and wrist *ain. #hat is the diagnosis$
a% tem*oral artritis
"% A
c% *olymyositis
d% sarcoidosis
207. A lady with *tosis/ *roximal muscle wea&ness. #hat is the "est test$
a% tensolin test
209. Question on Cushing syndrome
a% dexamethasone test
20<. Question on a lady with *seudo dementia
MCQ, P,@C)=A+@
1% Which of the following has the least cholenergic effect(repeated
question)**
a) Imipramine
"5 Prozac (fluexitine)
c) nortriptyline
d5 duxepine
e) amitryptaline
2- IN B!BI"#!"$ WI"%&!W'( W%I)% *+ "%$ +*''*WIN, IN&I)"$- "%$ W*!-"
P!*,N*-I- (!$P$"$&)
a5 con.ulsions
b) hyperthermia
c5 slurred speech
d) anxiety and irritability
e5 so/nolence
0- which of the following does not cause dependence
a) barbiturates
"5 diazepa/
7.
c) buspirone (buspar)
d5 /ethadone
e) alprazolam (xanax)
4- 8 years old boy very clean with very nice clothes, he is meticulous,
started to become anxious with palpitation and apprehension
frequently particularly before exams and at stress times. What is
your dianosis! (repeated)
a5 o1sessi.e co/pulsi.e disorder
b) super anxious disorder
c5 schizophrenia
d) sexual abuse
e5 none of the a1o.e
0% 2oung patient 1rought to hospital3 he was inaccessi1le for exa/ination
and has /utis/4 What is 5our diagnosis6
a) depression
"5 schizophrenia
c) hypomanic
d5 drun7
e) anxious
6- "emale chronic schizophrenic on medication. #ame with abnormal
movement in the face involvin bucco$oral muscles. What to do
next!
a5 -top the phenothiazines
b) decrease the dose of the phenothiazines
c5 increase the dose of the phenothiazines
d) %top phinothiazines and ive antipar&insonian drus.
e5 -tart antipar7insonian drugs
)ont4 ps5 /cqs
7% Which of the following will 1enefit /ost fro/ tric5clic anti-depressants
a) youn man with history of social problems
"5 /ale 89 5s with features of depression
c) male with features of schizophrenia
d5 fe/ale with a relati.e who has /a:or depressi.e episode
e) female who feels uilty and unhappy
8- 'll of the followin are ris& of suicide except
a5 fe/ale gender
b) male ender
c5 age
d) depression
e5 ph5sical illness
<% in transsexualis/ 5ou get
70
a) ood ender identity
"5 interest in the fe/ale sex
c) thin& of self as a man
d5 histor5 of cross dressing in childhood
e) history of child sexual abuse
1- in transvestism you find all of the followin except
a5 the5 are /ales 15 genot5pe
b) they are feminine and show interest in the other sex
c5 the5 are fe/inine and show no interest in the other sex
d) li&e to thin& of self as female
e5 nor/al childhood and upraising
;;-all of the following is true for huntington<s chorea $=)$P">
a) onset in mid sixties
"5 autoso/al do/inant
c) proressive disease
d5 associated with depression
e) associated with dementia
()$which of the followin has cross$tolerance with diazepam
a5 1ar1iturates
b) cocaine
c5 alcohol
d) haloperidol
e5 has no cross tolerance
;0-the /ost dangerous withdrawal s5/pto/ can 1e expected fro/
a) diazepam
"5 clonazepa/
c) chlordiazepoxide
d5 lorazepa/
e) oxazepam
cont. psy mcqs
(*$ an alcoholic man who was admitted to the hospital because of sha&in,
increased pulse rate, sweatin, etc. he was also disoriented and with visual
hallucination. +he most helpful medication is
a5 haloperidol
b) chlordiazepoxide(dose ),$,-m) in I#.
c5 chlordiazepoxide (dose 28-89/g) in the general ward
d) chlorpromazine
e5 pheno1ar1itone
;8- ffect is distur1ed in all of the following except
a) /ysthymia
72
"5 %e1ephrenic schizophrenia
c) 0ipolar disorder
d5 /elancholia
e) paranoid delusional psychosis
(1$ a man with both history of depersonalization and derealisation for
23() what do you thin&
a5 nor/al finding
b) ive anxiolytic
c5 needs urgent ad/ission to ps5chiatric ward to 7eep hi/ fro/
har/ing hi/self
d) needs neuroloical examination before anythin
e5 start ps5chotherap5
;?-a wo/an co/plains of sleep distur1ance( .arious aches and pain( ina1ilit5
to experience usual e/otions- (including grief( :o5( and pleasure)( and she
also co/plaining of loss o interest4 %er fa/il5 notices that she 1eca/e
dull( forgetful4 *n inter.iewing her she has decreased concentration( and
i/paired /e/or54 What is 5our diagnosis6
a) 'lzheimer disease
"5 pseudode/entia
c) multiple personality
d5 generalized anxiet5 disorder
e) severely depressed
(8$ 4,ys man, hypertensive on methyldopa, diuretics and 5#l
supplement, benzotropine, and vitamins (includin vitamin 6), he
was brouht to the hospital. +he nurse said that he was complainin
of sleep disturbance for the last two days, and impaired memory, and
he was brouht now because of aitation. 736 he was disoriented,
and confused, which dru is responsible for his confusional state!
a5 /eth5ldopa
b) vitamine 6
c5 1enzotropine
d) diuretics
e5 none of the a1o.e
;@- which of the following conditions( ps5chotherap5 not indicated>
a) phobic disorder
"5 seasonal affecti.e disorder
c) paranoid schizophrenia
d5 organic 1rain s5ndro/e
e) bipolar disorder
)-$ all of the followin drus can cause depression except
77
a5 propranolol
b) chlorpromazine
c5 ci/itedine
d) levodopa
e5 /eth5ldopa
2;-the /ain treat/ent of schizophrenia is
a) psychotherapy
"5 1eha.ioral therap5
c) neuroleptics
d5 fa/il5 therap5
e) isolation into controlled environment
))$ 'voidant personality disorder characterized by
a5 Perfectionis/ and inflexi1ilit5( which /a5 interfere with tas7
co/pletion
A5 )@PE,EH,=+=(=+@ +! E(A':A+=!H/ PE!CC:P=E6 #=+) EH(@/ AH6 GAH6=!,E
,EH,E !E ,E'E%=MP!+AHCE. +)E@ )A(E +EH6EHC@ +! C!MP'A=H !E M:'+=P'E
,!MA+=C ,@MP+!M,
c5 -ensiti.e to criticis/4 unwilling to get in.ol.ed with people unless
certain 1eing is li7ed
d) allows others to ma&e important decisions, and arees with
people to avoid re8ection
e5 insta1ilit5 of /ood( interpersonal relations ( and self i/age( also
i/pulsi.e 1eha.ior
20-ll of the following can 1e used on the treat/ent of 1uli/ia ner.osa except
a) antidepressants
"5 $)"
c) 0ehavioral therapy
d5 )ogniti.e therap5
e) (a 9b)
)*$ all of the followin can be used in the treatment of acute mania except
a5 'ithiu/
b) 6#+
c5 "razodone
d) :aloperidol
e5 )ar1a/azapin
28- &ifference 1etween schizophrenia and /ania is
a5 a1nor/al ps5cho/otor acti.it5
"5 delusion of grandiose
c5 thought 1roadcasting
79
d5 delusion of persecution
e5 hallucination
2A- the following drugs can cause /ania except (repeated)
a5 phenelzine
"5 trazodone
c5 car1a/azipine ("egraol)
d5 corticosteroids
e5 /eth5lphenidate
2?- new1orn *B$ wt C249 7g fullter/( /icrociphal5( shortened palpe1ral fissure(
/idface h5poplasia( a1nor/al pal/ar ceases( cardiac defect( and :oint
contractures4 "he /other said that she has a child with /ental retardation4 2ou
thin7 that the /other is a drug a1user4 What is the /ost co//on cause of drug
induced teratogenesis>
a5 &ieth5l stil1istrol (&$-)
"5 "halido/ide
c5 Phen5toin(fetal h5datoin s5ndro/)
d5 lcohol (fetal alcohol s5ndro/e)
e5 )ocaine
2D- child 1rought to $!( 5ou suspect sexual a1use( the first step in the Ex
a5 Notif5 child protecti.e agenc5
"5 d/it to hospital
c5 &iscuss the pro1le/ with the /other
d5 contact the police
e5 collect e.idence of a1use
2@- +light of ideas is /ost co//on in
a5 paranoid disorders
"5 dou1le depression
c5 /anic ps5chosis
d5 /elancholia
e5 deliriu/
09- FI ha.e alwa5s 1elie.ed in the good of /an 7ind 1ut I a/ not a wo/an 1ecause I
ha.e an da/Gs appleH said 15 the patient 5ou are inter.iewing( this is
a5 flight of ideas
"5 ideas of reference
c5 intellectualization
d5 tangentiall5
e5 loosing of association
0;- ll of the following are paranoid delusions
a5 persecutor5 delusions
"5 grandiosit5 delusions
c5 delusion of guilt
d5 so/atic delusions
e5 delusion of :ealous5
7<

02- depressed patient with second-degree heart 1loc74 "reat/ent
a5 i/ipra/ine
"5 a/itriptaline
c5 doxepine
d5 $)"
e5 None of the a1o.e
00- in lithiu/ toxicit5 what is true6
a5 pol5uria is a sign of intoxication
"5 h5poth5roidis/ is an indication to discontinue the drug
c5 can cause congenital heart ano/al5 in pregnant wo/an
d5 needs urgent ad/ission
0I- the ris7 of suicide is high in patients
a5 5ounger than ;?
"5 pre.ious suicidal atte/pt
c5 with /alignanc5
d5 elderl5 wo/an
e5 fe/ale with social circu/stances
08- treat/ent of o.erdose of anticholinergic drugs is
a5 ph5sostig/ine
"5 nalaxone
c5 flu/azenil
d5 respiredol
e5 he/odal5sis
0A- a1solute contraindication in $)"
a5 heart 1loc7
"5 EI two wee7s ago
c5 Intra-cranial lesion
d5 %istor5 of grand/al seizures
e5 Increased I)P
0?- delusion in de/entia is usuall5
a5 of grandiose
"5 though 1roadcasting
c5 of reference
d5 prosecuti.e in content
e5 thought insertion
0D- ll of the following is side effect of ph5nothiazine( except
a5 decreased sweating
"5 d5stonic
c5 orthostatic h5potension
d5 par7insonian s5ndro/e
e5 diarrhea
0@- People forget a1out the date and place of the experience 1ut can re/e/1er the
experience itself4 &x
a5 nor/al forgetfulness
9>
"5 wernic7eGs encephalopath5
c5 Jit B; deficienc5
d5 I/pending lzhei/erGs disease
e5 $arl5 de/entia
I9- &epersonalization is seen in
a5 schizophrenia
"5 affecti.e neurosis
c5 1ipolar /ania
d5 nor/al persons
e5 a(1(c(
I;- what is the 1est /anage/ent in a child with school pho1ia6
a5 change school 1ecause he /ight ha.e 1een 1ullied in the old school
"5 gi.e anxiol5tics
c5 send 1ac7 to school i//ediatel5
d5 $)"
e5 Nor/al in children
I2- which is seen in narcoleps5
a5 cataplex5
"5 sleep paral5sis
c5 h5pnogogic hallucinations
d5 irresisti1le sleep
e5 all of the a1o.e
I0- a patient with i/ipra/ine intoxication what is 5our first /anage/ent in $!6
a5 $), /onitoring ( 1ecause of itGs effect on the heart
"5 -top /edication
c5 ,i.e ph5sostig/ine
d5 d/it to hospital then do an5 thing else
e5 )all ps5chiatric on call
II- which one of the following patients 1enefits /ost fro/ an antidepressant6
a5 IA 5o fe/ale with low /ood( poor sleep K loss of Wt
"5 88 5o /ale with low /ood ( good sleep Kloss of Wt
c5 08 5o /ale with low /ood
I8- In the case of postpartu/ ps5chosis all of the following are true except
a5 treated with neurol5ptics
"5 usuall5 treated with antips5chotics
c5 can cause residual effect
IA- Which of the following antidepressants is less sedati.e K less anticholinergic6
a5 i/ipra/ine
"5 trazodone
c5 sertraline
d5 a/itript5line
e5 nortript5line
91
I?- patient has 1een treated after ta7ing ps5chosti/ulant( 5ou will ad.ice hi/ with
which of the following to /ini/ize co/plication6
a5 &iazepa/
"5 Eethadone
c5 &isulfira/
d5 follow up in support group
ID- Which of the following is least in fa.or of schizophrenia
a5 +lat affect
"5 &eterioration of social K occupational function
c5 Brief recurrent attac7 of ps5chosis
d5 Bizarre delusions
e5 %allucinations
I@- patient( who is char/ing( superficiall5 /anipulati.e( doesnGt learn fro/ his
pre.ious experiences and lac7 of re/orse4 What t5pe of personalit5 does he
ha.e6
a5 %istrionic
"5 nti-social
c5 )o/pulsi.e
d5 Borderline
89- in con.ersion disorder( all of the following can 1e seen except
a5 la 1elle indifference
"5 unconsciousness with s5/pto/s
c5 defense /echanis/ of pro:ection and intellectualization
8;- I2 5o /ale patient with %B* short ter/ 1eha.ioral changes( loss of /e/or5( on
da5 of ad/ission he de.eloped con.ulsion( which of the following in.estigation
5ou should order
a5 )" scan
"5 E!I
c5 Blood for toxic screen
d5 Blood for %IJ test
82- all of the following drug withdrawal can cause con.ulsion except
a5 diazepa/
"5 opiate
c5 1ar1iturate
d5 larozepa/
80- all of the following are associated with child a1use except
a5 low social class
"5 parents are .icti/s when the5 are 5oung
c5 parents are ps5chotics
d5 ho/osexualit5
8I- a1used child presented in the $! 5ou will do all of the following except
a5 collect child cloths
92
"5 exa/ine the upper li/1
c5 collect sa/ples for gonorrhea
d5 .aginal sa/ples for sper/atozoa
e5 exa/ine the .agina for possi1le laceration
88- the natural histor5 of attention deficit s5ndro/e is
a5 /a:orit5 of 7ids grow out their difficult5 when the5 reach adulthood
"5 89L of 7ids ha.e cri/inal histor5 when the5 1eco/e adult
c5 /a:orit5 of 7ids canGt grow out their difficult5 which persist during the
adulthood
d5 ;8L of 7ids ha.e e.idence of neurological defect
8A- +e/ale recentl5 di.orced presented with dizziness( retrosternal chest pain
radiating to 1oth ar/s K occassional difficult5 in swallowing4 What is the /ost
li7el5 diagnosis6
a5 esophageal spas/
"5 panic attac7
c5 achalasia cardia
d5 reflux esophagitits
e5 angina pectoris
8?- Jisual hallucination /ost co//onl5 caused 15
a5 deliriu/ tre/ans
"5 schizophrenia
c5 organic 1rain disease
d5 /ood disorder
8D- the nor/al de.elop/ent of I-8 5o child includes all of the following except
a5 the de.elop/ent of conscience
"5 the de.elop/ent of a1stract thin7ing
c5 learning the sex distinction
d5 acquiring an accepta1le degree of initiati.e
e5 acquiring the capacit5 for cooperati.e pla5
8@- a 29 5o fe/ale who is a1out de.eloping huntingtonGs chorea( her father and grand
father had the disease and her uncle as well( what is the chance that she will
contract the disease6
a5 89L
"5 ;99L
c5 28L
d5 9L e) ?8L
A9- delusion disorder is
a5 unrealistic fixed 1eliefs that cannot 1e changed
"5 thought 1roadcasting
c5 false perception of actual sti/uli
A;- in order to /a7e a diagnosis of d5sth5/ic disorder
a5 depressed /ood for /ore than 2 5ears
9-
"5 depressed /ood for /ore than 2 /onths
c5 depressed /ood for /ore than 2 wee7s
d5 depressed /ood for /ore than 2 da5s
e5 none of the a1o.e
A2- loosing of association can 1e found in
a5 /ania
"5 schizophrenia
c5 dissociati.e disorder
A0- the defense /echanis/ of pho1ia pri/aril5 includes
a5 displace/ent
"5 a.oidance
c5 s5/1olization
d5 rationalization
AI- $lderl5 lad5 li.ing alone( 1rought 15 her daughter 1ecause she fall occasionall5(
her de/entia was pre.enta1le if treated earl5( what is this condition6
a5 pernicious ane/ia
"5 huntingtonGs disease
c5 WilsonGs disease
d5 lzhei/erGs disease
A8- patient co/plains of lanciating pain during sha.ing of his face4 What drug will he
1enefit fro/6
a5 propranolol
"5 car1a/azapine
c5 steroid
d5 phen5toin
e5 halopeidol
AA- the i//ediate treat/ent of an acute ps5chotic reaction to an un7nown drug is6
a5 "hioredazine
"5 "rifluperazine
c5 )hlorpro/azine
d5 %aloperidol
e5 &iazepa/
A?- 88 2* /an is 1rought to the $! 1ecause of recent alar/ing change in his
personalit5( he has 1een acting strangel5( telling :o7es at the wrong ti/e and
place( showing increasingl5 poor :udge/ent and expressing ideas of grandiosit54
Ph5sical exa/ination and 1lood serolog5 is nor/al4 What is 5our diagnosis6
a5 /ultiple sclerosis
"5 late onset schizophrenia
c5 h5po/anic state
d5 presenile de/entia
AD-which state/ent a1out delusions is incorrect6
a5 the5 occur onl5 in ps5chosis
"5 the5 /a5 arise secondar5 to other co/plaint
c5 the5 cannot 1e changed 15 reason
d5 the5 can 1e persecutor5 in content
9.
e5 in depression the content fits the /ood
A@- all of the following are considered /odes of pro.en efficac5 in the treat/ent of
schizophrenia except
a5 "hiothixene
"5 $)"
c5 %aloperidol
d5 Niacina/ide
e5 "rifluperazine
?9- patient goes to 1ed at ;9>99PE and wa7es up at 2>99 E and cannot go to 1ac7
to sleep4 "his sleep pattern recurs e.er5 night( 5our diagnosis is>
a5 nxiet5 neurosis
"5 Narcoleps5
c5 Paranoid schizophrenia
d5 &epression
e5 *1sessi.e co/pulsi.e disorder
?;- ll of the following is side effect of i/ipra/ine except
a5 dr5 /outh
"5 1lurred .ision
c5 increased sali.ation
d5 par7insonian reaction
e5 constipation
?2- which of the following characteristics would 1e helpful in differentiating attention
deficit h5peracti.it5 disorder fro/ conduct disorder in children6(repeated)
a5 resistance to discipline
"5 te/per tantru/
c5 distracti1ilit5
d5 aggressi.eness
e5 truanc5
?0- Which of the following is a correct state/ent a1out cosleeping (children sleeping
with their parents during the night)6
a5 )hildren who sleep with their parents are /ore li7el5 to sleep through
the night
"5 )osleeping encourages independence and autono/5 in the child
c5 )osleeping is less co//on in 1lac7 fa/ilies than in )aucasian fa/ilies
d5 Eost pediatricians support cosleeping in the preschool child
e5 Eost toddlers sleep with a parent at least once a /onth
?I- which of the treat/ent is /ost li7el5 to 1e helpful for attention deficit h5peracti.it5
in a ;9 2* child
a5 Beha.ior /odification
"5 Biofeed1ac7
c5 %5pnosis
d5 Eodified diet
e5 Jision training
90
?8- a co//on side effect of serotonin re-upta7e inhi1itors is
a5 increased appetite and weight gain
"5 dr5 /outh and constipation
c5 cardiac conduction defect
d5 sexual d5sfunction
e5 postural h5potension
?A- which of the following state/ents is correct a1out alcohol withdrawal seizure6
a5 the5 occur I-? da5s after drin7ing has 1een stopped
"5 the5 can 1e pre.ented 15 proph5lactic treat/ent with thia/in
c5 the5 are usuall5 the first s5/pto/ or deliriu/ tre/ens
d5 the patient will require long ter/ treat/ent with anticon.ulsant
e5 the5 can 1e pre.ented 15 sti/ulation of the ,B(ga//a a/ino
1ut5ric acid) receptors in the 1rain
??- I0 2* /an ad/itted for e/ergenc5 gastrecto/5 de.eloped confusion on the
third post-operati.e da54 %e co/plains of lac7 of sleep due to cats in his hospital
roo/ all night and co/plains also of coc7roaches on the ceiling4 %e is noted to
1e flushed and tre/ulous 15 the nurses during the da54 "he /ost li7el5 pro1le/
is>
a5 Post-operati.e electrol5te i/1alance
"5 Paranoid schizophrenia
c5 &epressi.e ps5chosis
d5 &eliriu/ tre/ens
e5 noxic 1rain s5ndro/e
?D- patient with anorexia ner.osa>
a5 Belie.es deep inside that she is underweight
"5 #suall5 a.oids excessi.e exercise
c5 Will pro1a1l5 not resu/e nor/al /enses when she reco.ers
d5 Eeasures success in ter/ of not eating and weight loss
e5 *ften de.elops purging later in the course of the illness
?@- which of the following is correct a1out child a1use
a5 )hildren donGt lie a1out sexual or ph5sical a1use
"5 "he parents were often .icti/s of a1use the/sel.es
c5 It occurs is usuall5 so/e1od5 the child does not 7now
d5 It occurs /ainl5 in the lower socio-econo/ic class
e5 2ou should not report 5our suspicions to the )hildrenGs id -ociet5
unless there is a/ple e.idence of a1use
D9- which of the following is correct a1out depression in children6
a5 +a/il5 therap5 should 1e a.oided 1ecause it scapegoats the child who
is alread5 .ulnera1le
"5 -5/pto/s /a5 /anifest as antisocial 1eha.ior
c5 ntidepressants generall5 are not effecti.e in children
d5 "he suicide rate in children aged D-;0 is higher than it is in older
adolescents
e5 &epression in children has 1een shown to prodro/e to the later
de.elop/ent of schizophrenia
92
D;- 8I 2* /an has 1eco/e forgetful( preoccupied( withdrawn( suspicious and
dishe.eled4 %is ph5sical exa/ination was nor/al4 "he patient had 1een with his
co/pan5 for twent5-two 5ears and was considered an excellent e/plo5ee4
Which of the following is the /ost li7el5 diagnosis6
a5 Eulti infarct de/entia
"5 %5poth5roidis/
c5 -chizophrenia
d5 lcoholis/
e5 lzhei/erGs
D2- which of the following is the 1est treat/ent for agorapho1ia with panic6
a5 'orazepa/ ;/g tid
"5 +luxetine 29/g od
c5 +luphenazine 29/g od
d5 )ogniti.e therap5
e5 'ithiu/ 009/g qid
D0- which of the following exe/plifies a t5pe of thought that is s5/pto/atic of
schizophrenia6
a5 "he tele.ision sending su1li/inal /essages
"5 "he foul odor e/anating fro/ the .entilation s5ste/ (in order to poison
the indi.idual)
c5 "houghts that :u/p fro/ topic to topic
d5 "he fear of heights
e5 ,od is tal7ing to the indi.idual
DI- $)" is useful in the treat/ent of>
a5 &epersonalization s5ndro/e
"5 Eania
c5 )on.ersion disorder
d5 )hronic schizophrenia
e5 &5sh5/ic disorder
D8- 28 2* surgical nurse is concerned he is losing his /ind4 +or the past A /onths
heGs 1een preoccupied with conta/ination on his ward4 %e has 1een a.oiding
touching patients( door7no1s( etc4( and has 1een washing his hands excessi.el54
Which of the following treat/ent is the /ost li7el5 to decrease his preoccupation
and his hand washing6
a5 +luoxetine
"5 'orazepa/
c5 Perphenazine
d5 Insight-oriented ps5chotherap5
e5 Nifedipine
DA- which of the following is the least effecti.e in the treat/ent of schizophrenia6
a5 +luspirilene
97
"5 Phenelzine
c5 "hiothixene
d5 !esperidone
e5 "hioredazine
D?- /ale patient is reco.ering fro/ an episode of /a:or depression4 +i.e 5ears ago
he had si/ilar episodes which resol.ed spontaneousl5 after eight /onths4 +or
this patient( which of the following state/ents is correct6
a5 %e will li7el5 ha.e episodes less frequentl5 in the future
"5 "here is a I9L chance he will de.elop 1ipolar /ood disorder in the
future
c5 "here is a ;8L chance he will e.entuall5 7ill hi/self
d5 %e is at increased ris7 of de.eloping lzhei/erGs disease
e5 %is antidepressant /edication should 1e discontinued shortl5 after his
s5/pto/s re/it to pre.ent a switch into /ania
DD- which of the following is essential in /a7ing then diagnosis of schizophrenia
a5 %allucinations
"5 &elusions
c5 &eterioration in function
d5 &isorganized thin7ing
e5 *nset 1efore age 8I
D@- 2A2* student has 1een well controlled on lithiu/ for fi.e 5ears4 %is last
hospitalization was for a se.ere ps5chotic depression4 %e is now co/plaining of a six-
wee7 histor5 of d5sphoria( low energ5( lac7 of dri.e and sleepiness
2our /ost appropriate initial inter.ention should 1e>
a5 -top lithiu/
"5 &o th5roid function test
c5 -tart an antidepressant
d5 -tart an antidepressant and neuroleptic
e5 )hic7 his he/oglo1in
@9- in a 0 2* child( each of the following would 1e descri1ed as nor/al 1eha.ior
except>
a5 Eastur1ation
"5 "hu/1 suc7ing
c5 ttach/ent to an inani/ate o1:ect such s a stuffed ani/al
d5 Inconsola1le distress when separated fro/ /other
e5 )an run easil5 and negotiate stairs alone with alternate feet
@;- each of the following state/ents a1out delusional disorders is correct $=)$P"
a5 "he5 are less co//on in schizophrenia
"5 "he patient often functions well at wor7
c5 It is unli7el5 the patient will go on to de.elop schizophrenia
d5 "he delusion responds well to neuroleptics
e5 &elusions are ne.er 1izarre
99
@2- in the treat/ent of repetiti.el5 suicidal uni.ersit5 student( who has a personalit5
disorder( each of the following would 1e reco//ended except>
a5 Brief hospitalization
"5 Phar/acotherap5
c5 Ps5choanal5sis
d5 Eo1ilization of social supports
e5 Beha.ior therap5
@0- concerning childhood ps5chiatric disorder( each of the following state/ents is
correct $=)$P">
a5 )hildren with /ild /ental retardation are identifia1le at age 2-0 5ears
"5 )hildren with de.elop/ental reading disorder ha.e nor/al intelligence
c5 "he presence of autis/ in si1lings of autistic children is 89 ti/es that in
the general population
d5 "he diagnosis of encopresis can 1e /ade when a child is I2*
e5 )hildren with gender identit5 disorder rarel5 go on to de.elop
transsexualis/ in adult life4
@I- in the use of /eth5lphenidate in children( side effects include each of the following
$=)$P">
a5 -leeplessness
"5 Weight gain
c5 Itch5 s7in
d5 %5peracti.it5
e5 ,rowth retardation
<0% Concerning an adolescent who is fre4uently truant/ sexually ex*loitive/ lies/ steals/
and a"uses drugs/ each of the following statements is correct E1CEP+B
a5 "here is a good chance he has a reading disa1ilit5
"5 %e li7el5 to ha.e no re/orse or guilt
c5 %e /a5 ha.e soft neurological signs
d5 %e is li7el5 isolated fro/ his peers
e5 %e should recei.e intensi.e insight-oriented indi.idual ps5chotherap5
@A- which of the following is correct regarding lzhei/erGs disease
a5 It occurs in ;99L of cases of down s5ndro/e
"5 It often first presents as sociall5 inappropriate 1eha.ior
c5 It is /ore co//on in )aucasians
d5 It accounts for D8-@9Lof all de/entia in the elderl5
e5 It is characterized 15 a step-wide deteriorating course
@?- each of the following state/ents a1out ps5chological disorder is correct $=)$P">
a5 &epression is a1out three ti/es as co//on in adolescents as in
children
"5 ntisocial 1eha.ior( deteriorating school perfor/ance and
h5pochondriasis are unusual s5/pto/s in depressed children
c5 Eedication is useful in the treat/ent of the child
d5 "he /ain conflict in a child with ps5chneuorosis is 1etween the parents
e5 nxiet5 disorders in children rarel5 persist( as the children rapidl5
de.elop new defense which change the for/ of the neurosis
9<
@D- which of the following is helpful sign in differentiating attention deficit h5peracti.it5
disorder fro/ conduct disorder in children6
a5 *1stinac5 and resistance to discipline
"5 +requent tantru/s
c5 &istracti1ilit5
d5 ggression
e5 "ruanc5
@@- which of the following is correct to anorexia ner.osa
a5 -ignificant depression usuall5 occurs
"5 ppetite is not initiall5 lost
c5 Pursuit of thinness 1eco/es the onl5 priorit5
d5 'oosing weight 1eco/es associated with an escalation of anger
e5 Purging often de.elops later in the course of the illness
;99- A? 2* retired /ale teacher has 1een charged 15 police with indecent assault
on an eight 2* girl4 "here is no pre.ious arrest record or ps5chiatric histor54 %is
wife reports decreased self-care( increased irrita1ilit5 and forgetfulness in the
past fi.e /onths4
Which of the following is the /ost li7el5 diagnosis6
a5 Pedophilia
"5 %5po/anic episode
c5 &epression
d5 lzhei/erGs
e5 Personalit5 disorder
;9;- which of the following is a correct state/ent a1out suicide6
a5 It is /ost co//on cause of death in the 29-09 age range
"5 It is one quarter as co//on as suicidal atte/pts
c5 Jicti/s seldo/ gi.e warning of their intentions
d5 It occurs in 8-;9 L of patients suffering fro/ recurrent /a:or depression
e5 It occurs in 8-;9L of patients suffering fro/ schizophrenia
;92- I92* presents to a ps5chiatrist co/plaining of depression( cr5ing spills(
inso/nia( anorexia and weight loss following the death of her hus1and two
/onths ago4 -he is also concerned she is losing her /ind 1ecause she
occasionall5 has a feeling that dead hus1and is present in the roo/ when she
atte/pts to sleep at night4 In fact( at ti/es she can .aguel5 see his in the roo/(
although she 7nows he is not reall5 there4 Which of the following is the /ost
li7el5 diagnosis6
a5 Post trau/atic stress disorder
"5 Ea:or depression with ps5chotic features
c5 Nor/al grief reaction
d5 ,eneralized anxiet5 disorder
e5 $arl5 o1sessi.e co/pulsi.e disorder
<>
;90- which of the following state/ents is correct with respect to infantile autis/6
a5 It is /ore co//on than huntingtonGs chorea
"5 1out half of these children show s5/pto/s of an organic 1rain
s5ndro/e
c5 "he incidence is highest a/ong fe/ales of ash7enaz5 Mewish origin
d5 &espite dela5s in language de.elop/ent( intelligence is usuall5 nor/al
e5 It is /ore pre.alent in the lower socio-econo/ic classes
;9I- which of the following is correct with respect to agorapho1ia with panic attac7s
a5 *ccurs with equal frequenc5 in /en and wo/en
"5 #suall5 has itGs onset after age 08
c5 !uns in fa/ilies
d5 !esponds poorl5 to treat/ent with phenelzine
e5 *ccurs /ore frequentl5 in indi.iduals with histrionic personalit5
disorder
;98- which of the following is least effecti.e in the treat/ent of the acute s5/pto/s of
-chizophrenia6
a5 +luspirilene
"5 Phenelzine
c5 Perphenazine
d5 "hiothixene
e5 )lozapine
1>2% withdrawal s5ndro/e /a5 occur after the discontinuation of each of the
following $=)$P">
a5 &iazepa/
"5 '-&
c5 Nicotine
d5 )hlordiazepoxide
e5 )affeine
1>7% 0 5ear-old child has ta7en off her panties se.eral ti/es at nurser5 school4
What is the /ost li7el5 explanation for this6
a5 -exual a1use at ho/e
"5 Pin wor/s
c5 Nor/al 1eha.ior
d5 .aginal infection
e5 difficulties in her oedipal phase
1>9% In e.aluating the ris7 of lea.ing a1used children with parents4 Which of the
+ollowing is a good prognostic factor6
a5 .er5 5oung child (i4e4 an infant who is not a1le to crawl or wal7)
"5 Parents willing to enter /arital therap5
c5 Intelligent parents
d5 "he a1sence of histor5 of a1use in the parentsG own childhood
e5 "he a1usi.e parent is e/plo5ed in law enforce/ent and is /ore li7el5 to
adhere to rules and expectations4
1><% Which of the following is a correct state/ent a1out the treat/ent for
depression
In the elderl5
<1
a5 "ric5clic antidepressants should 1e a.oided due to their strong
anticholenergic side effect
"5 "hose with pseudode/entia will clear with treat/ent and ha.e the sa/e
ris7 of de.eloping de/entia as others in the population
c5 &epression responds well to treat/ent and has prognosis si/ilar to that
of 5ounger patients
d5 $)" is generall5 safer than /edication
e5 Ps5chotherap5 should 1e a.oided due to the ris7 of stirring up underl5ing
fears and feelings a1out death
11>% +ollowing an argu/ent with her 1o5friend of three wee7s( a ;A 2* girl too7
an
*.erdose of aspirin4 -he was ta7en to the general hospital and ad/itted to
the ps5chiatric unit4 +our da5s later she threatened to punch out another
girl on the unit after a .er1al insult4 *n the da5 5ou inter.iew her she is
hea.il5 /ade up( wearing tight spandex clothing and is despondent and
angr54 -he clai/s she has 1een Freall5 depressedH since the argu/ent with
her 1o5friend4
Which of the following would 1e '$-" helpful6
a5 ,roup therap5
"5 Indi.idual supporti.e therap5
c5 +a/il5 histor5
d5 ntips5chotic
e5 ntidepressant
111% A2 2* wo/an is ad/itted to a /edical unit 1ecause of a 29l1 wt loss
o.er the
Pre.ious two /onths4 -he also reports anorexia( fatigue and se.er
constipation4
Ph5sical exa/ination( 1lood wor7 and sig/oidscop5 are all nor/al4
Which of the following would 1e the /ost appropriate /anage/ent6
a5 )ontinuing ph5sical in.estigations to rule out organic causes
"5 /itriptline
c5 'orazepa/
d5 %igh fi1er diet
e5 &esipra/ine
112% 08 2* wo/an presents with the recent 1elief that since Ni/ )a/p1ell
was defeated she /ust ta7e o.er as Pri/e Einister of )anada4 -he
requires two hours of sleep per night( and feels wonderful and energetic4
Which of the following s5/pto/s is she '$-" li7el5 to show6
a5 uditor5 hallucinations
"5 &isorganized thin7ing
c5 &isorientation for ti/e and place
d5 $asil5 angered and irritated
e5 Poor insight
<2
11-% 08 2* oncologist has a I wee7 histor5 of feeling sad( irrita1le( cr5ing(
inso/nia( ;9 l14 wt loss( withdrawn fro/ wor7 and friends and the 1elief
that he has caught leu7e/ia fro/ one of his patients4 %e has also 1een
hearing a .oice telling hi/ to 7ill hi/self4
Which of the following will 5ou use first to treat hi/6
a5 E* inhi1itors
"5 ntips5chotic drugs plus tric5clic antidepressants
c5 "ric5clic antidepressant
d5 $)"
e5 'ithiu/ Otric5clic antidepressants
11.% Which of the following is a correct state/ent a1out neurol5ptic therap5 for
schizophrenia6
a5 It wor7s onl5 ?8L of the ti/e
"5 It pre.ents secondar5 depression
c5 It re.erse the a/oti.ational state
d5 It should 1e the sa/e as the initial dose when used for /aintenance
e5 It can worsen hallucinations and delusions in 8L of cases
110% In treating /anic episodes( which of the following is /ost useful in the first
three da5s of treat/ent6
a5 %aloperidol 8 /g 1d
"5 )hlorpro/azine ;9 /g po 1d
c5 'ithiu/ 099 /g od
d5 )ar1a/azepine 299 /g 1d
e5 quit( di/l5-lit roo/ to decrease sti/ulation
112% Which of the following is /ost li7el5 to precipitate seizure6
a5 )lo/ipra/ine
"5 /itript5line
c5 &esipra/ine
d5 Eaprot5line
e5 "ran5lc5pro/ine
117% Which of the following is correct regarding school pho1ia6
a5 "he incidence pea7s at school entr5 and graduall5 droops off with
increasing age
"5 It is i/portant to do a fa/il5 assess/ent and fa/il5 therap5 prior to
returning the child to school
c5 "ric5clic antidepressants ha.e 1een useful
d5 ,et the child 1ac7 to school as soon as possi1le
e5 It often signifies underl5ing depression
119% Which of the following is correct with respect to alcoholic hallucinations6
a5 "he hallucinations are usuall5 auditor5
"5 It is 1est treated with 1enzodiazepines
c5 It usuall5 occurs in the chronic alcoholic who goes on a particularl5
hea.5 1inge
d5 It is usuall5 a chronic disorder
<-
e5 It is /ore co//on than schizophrenia
11<% Which of the following is the 1est treat/ent for agorapho1ia with panic6
a5 'orazepa/ 2 /g po prn( up to I/g od
"5 I/ipra/ine ;89/g po qhs
c5 Eeth5lphenidate ;9/g po qid
d5 )ogniti.e therap5
e5 Propranolo 29-D9 /g od
12>% which of the following is least effecti.e in the treat/ent of 1uli/ia ner.osa
a5 1eha.ior therap5
"5 ps5chotherap5
c5 appetite suppression therap5
d5 fa/il5 therap5
e5 tric5clic antidepressant
121% lcoholic patient presented to the hospital with ataxia( n5stag/ous(
a1ducent ner.e pals5 and /ild :aundice( li.er enlarged 15 8 c/ 1elow the
costal /argin4 What is the /ost li7el5 diagnosis6
a5 'i.er cirrhosis
"5 Werni7eGs encephalopath5
c5 Jiral hepatitis
d5 )ere1ellar disease
122% Which of the following drugs is the least sedati.e and anticholenergic
which can 1e prescri1ed safel5 to elderl5 patients with depression6
a5 +luxitine
"5 E* inhi1itor
c5 I/ipra/ine
d5 -ertraline
e5 "razodon
12-% Eale patient presented to hospital co/plaining of feeling hi/self unreal
and the word around hi/ li7e dra/ for two 5ears duration4 "here is no
histor5 of hallucination ps5chotic attac7s4 What is 5our diagnosis6
a5 -chizophrenia
"5 &issociati.e disorder
c5 Eood disorder
d5 %5sterical neurosis
12.% Nor/al grief reaction can include all of the followings $=)$P">
a5 +eeling of guilt
"5 -uicidal ideation
c5 Eore sleep periods
d5 'ess eating
e5 cceptance
120% ll of the following state/ent a1out orgas/ are true except>
<.
a5 as satisf5ing if achie.ed 15 /astur1ation co/pared to coitus
"5 it ta7e the aging fe/ale longer to reach orgas/ co/pared to a 5ounger
fe/ale
c5 si/ultaneous orgas/ should 1e a goal
d5 orgas/ is followed 15 resolution
e5 /an5 fe/ales are /ultiorgas/ic
122% )hronic schizophrenic patient( on antips5chotic and antipar7inson therap5(
de.elops tardi.e d5s7inesia4 What is the next step of /anage/ent>
a5 )ontinue antips5chotic and increase antipar7inson drug
"5 &ecrease and stop antips5chotic drug
c5 )ontinue an5ips5chotic and decrease antpar7inson drug
d5 Increase antips5chotic and stop antipar7inson drug
127% 00 2* patient with progressi.e de/entia presented to the hospital4 %e is
i//o1ile( /ute( 1ut alert4 What is the /ost li7el5 diagnosis6
a5 lzhei/erGs disease
"5 I&- co/plex de/entia
c5 -chizophrenia
d5 &5sthi/ia
e5 utis/
129% "he 1est /anage/ent of c5cloth5/ia>
a5 ps5chotherap5
"5 1eha.ior therap5
c5 lithiu/
d5 antidepressant
12<% 'oosing of association can 1e seen in>
a5 schizophrenia
"5 /ania
c5 dissociati.e disorder
d5 Personalit5 disorder
1->% "he differential diagnosis of stupor includes each of the following $=)$P">
a5 -chizophrenia
"5 %5steria
c5 Brain s5ndro/e
d5 nxiet5
e5 &epression
1-1% I92* /an has 1een on constant lithiu/ therap5 for four tears 1ecause of
a series of h5po/anic episodes in his twenties and earl5 thirties4 %e is
currentl5 recei.ing ;299/g4 *f lithiu/ car1onate dail54 "wice /onthl5
lithiu/ seru/ le.els ha.e re/ained 1etween 94D and ;49 /$qB'( and his
present le.el is 94@2/$qB'4 he is now co/plaining of a six-wee7 histor5 of
latherg5( drowsiness( depression and lac7 of dri.e4 2our /ost appropriate
initial /anage/ent would 1e>
a5 Initiate tric5clic antidepressant
"5 ,i.e ph5sostig/ine to reduce lithiu/ side effect
c5 &eter/ine the seru/ electrol5tes
d5 -top the lithiu/ i//ediatel5
e5 d/inister an antipar7insonian drug
<0
1-2% ;@ 2* 5outh presents with a histor5 of setting fire to his 1edroo/4 When
1rought to the hospital 15 the police( he is .er5 frightened 1ut quite hostile4
%e infor/s 5ou that a .oice is calling hi/ a PfaggotG and that is wh5 he set
the fire4 %e 1elie.es that the )B) news had a special /essage :ust for hi/4
2our pro.isional diagnosis is schizophrenia and 5our /ost fa.ored
differential diagnosis is>
a5 ffecti.e disorder
"5 /pheta/ine ps5chosis
c5 Bro/ide intoxication
d5 '-& ps5chosis
e5 )hronic /ari:uana a1use
1--% $ach of the following is characteristic of deliriu/ $=)$P">
a5 anxiet5
"5 la1ile /ood
c5 neologis/
d5 disorientation
e5 illusions
1-.% for/ulation in ps5chiatr5 is
a5 su//er5 of the case
"5 particular theoretical orientation of .iewing ps5chiatric
patient
c5 h5pothesis
d5 histor5
e5 n outdated concept no longer of use in ps5chiatr5
1-0% Which anxiol5tic treat/ent is /ost li7el5 to pro.e satisfactor5 for a patient
with ad.anced li.er disease
a5 *xazepa/
"5 &iazepa/
c5 )hlordiazepoxide
d5 )hloral h5drate
e5 +lurazepa/
1-2% )linical /anifestation of con.ersion disorders /a5 include each of the
following $=)$P">
a5 phonia
"5 &eafness
c5 /nesia
d5 Paraplegia
e5 Pain
1-7% In the course of sexual counseling ( patients are /ost li7el5 worried a1out>
a5 Eastur1ation
"5 I/potence
c5 -odo/5
d5 Incest
<2
e5 Nor/alit5
1-9% $ach of the following state/ents a1out ps5chological disorders is correct
$=)$P">
a5 &epression is a1out three ti/es as co//on in adolescents as in
the children
"5 ntisocial 1eha.ior( chronic energ5 loss( deteriorating school
perfor/ance and h5pochondriaGs are co//on depressi.e
equi.alents4
c5 !ecent studies confir/ that the use of /edication is useful in the
treat/ent of the depressed child
d5 "he pri/ar5 conflict in a child with ps5chonneurosis is 1etween the
child and parents
e5 nxiet5 disorders in children rarel5 persist( as the children rapidl5
de.elop new defense which change the for/ of the neurosis
1-<% If an adequate course of neurol5ptic dose not control a schizophrenic
ps5chosis then the next treat/ent of choice could 1e>
a5 nti-depressants
"5 second fa/il5 of neurol5ptics
c5 'ithiu/
d5 Eega.ita/ins
e5 Blood dial5sis
1.>% "he /ost effecti.e and econo/ical for of "he treat/ent for a disa1ling
pho1ia is>
a5 Einor tranquilizers
"5 Insight oriented ps5chotherap5
c5 "ric5clic antidepressants
d5 Beha.ioral therap5
e5 E*I
1.1% Which of the following is correct regarding wernic7eGs encephalopath56
a5 *ften de.elops into Norsa7offGs e.en when treated4
"5 It has /ortalit5 rate of ?9-D9L if untreated
c5 "he treat/ent is thia/in( ;99/g po dail5 = 8 da5s
d5 "he s5/pto/s include n5stag/us( ataxia and opisthotonus
e5 None of the a1o.e
1.2% 09 2* %IJ positi.e /an presents with s5/pto/s of apath5( depression
and a/oti.ation( and shows signs of ps5cho/otor retardation4 %e also has
decreased appetite( sleep distur1ance and low energ54 Which of the
following state/ents is correct a1out this situation6
a5 "he use of Q" is unli7el5 to i/pro.e the a1o.e s5/pto/
<7
"5 "ric5clic antidepressants should 1e a.oided due to their ineffecti.eness
in treating depression in %IJ positi.e indi.iduals
c5 "his patient /ost li7el5 has cogniti.e i/pair/ent
d5 "he s5/pto/s are pro1a1l5 a nor/al reaction to the stress of 1eing
%IJ positi.e and ps5chotherap5 is the treat/ent of choice
1.-% the toxic effect of lithiu/ includes each of the following $=)$P">
a5 slurred speech
"5 co/a
c5 constipation
d5 ataxia
e5 nausea and .o/iting
1..% In depression $)" wor7s 1est if >
a5 ntidepressant /edication is first used
"5 "he patient understands the procedure and 1elie.es it will wor7
c5 "he electrodes are applied 1ilaterall5
d5 /uscle relaxant is gi.en prior to the procedure
e5 &elusions and hallucinations are /ini/al
1.0% 09 2* 1eing treated for his first episode of schizophrenia with haloperidol
;9/g dail5 and 1enzotropine 2/g dail5 is still feeling ner.ous and restless
despite a reduction in his auditor5 hallucinations4
What would 1e 5our next step6
a5 Increase the dose of haloperidol
"5 &ecrease the dose of haloperidol
c5 )hange to different class of antips5chotics
d5 Increase the dose of the 1enzotropine
e5 &o supporti.e ps5chotherap5 to help decrease his anxiet5 associated
with stress of ha.ing such a serious illness
1.2% which of the following is correct concerning depression in the elderl5
a5 "ric5clic antidepressant do not wor7 as well as 5oung patients
"5 "hose with pseudode/dntia ha.e a li7elihood of ha.ing an underl5ing
real de/entia
c5 &epression responds well to treat/ent and has a prognosis si/ilar to
that of 5oung patients
d5 $)" is usuall5 the treat/ent of choice 1ecause it is safer than
/edication
e5 &epression is a nor/al part of aging process and is usuall5 due to the
deterioration of ph5sical and /ental 1ilities
1.7% )o//on side effects of antips5chotic drugs include each of the following
$=)$P">
a5 -lurred speech
"5 nxiet5 and restlessness
c5 Blurred .ision
d5 Nausea and .o/iting
e5 "re/ors
<9
1.9% Which of the following people is in the categor5 with highest rate of suicide
a5 5oung i/pulsi.e fe/ale
"5 di.orced /ale o.er I9 5ears of age with alcohol dependence
c5 5oung /ale with schizophrenia
d5 /iddle aged wo/an with /edical illness
e5 n un/arried /ale aged 29-09 5ears with drug a1use
1.<% n D-5ear-old 1o5 has a 1eha.ioral pro1le/ at school4 %e is o.erl5 acti.e(
not pa5ing attention( is disrupti.e and has poor concentration4 In his
treat/ent( which of the following /edications would 1e /ost useful6
a5 )ar1a/azpine
"5 'orazepa/
c5 Pi/ozide
d5 Eeth5lphenidata
e5 Propranolo
10>% Which of the following is the '$-" co//on pro1le/ referred to
ps5chiatric e.aluation of children6
a5 ggression
"5 Poor peer relationships
c5 "e/per tantru/s
d5 -chool failure
e5 $nuresis
101% Which of the following is not correct with regard to attention deficit
h5peracti.it5 disorder6
a5 Ps5chosti/ulants are /ore effecti.e than other drugs
"5 Not all children show excessi.e /otor acti.it5
c5 "he disorder rarel5 persists into adulthood
d5 significant nu/1er de.elop antisocial personalit5 disorder in
adulthood
e5 Poor e5e-hand coordination or other FsoftH neurological signs /a5 1e
present
102% Which of the following diagnosis is not co//onl5 associated with conduct
disorder6
a5 ttention deficit h5peracti.it5 disorder
"5 'earning disa1ilit5
c5 'ower IR
d5 *ppositionl disorder
e5 *ften persists into adulthood
10-% 09 2* /arried wo/an is finall5( after /an5 trials of /edication( well
controlled on lithiu/ ;299/g dail5 and haloperidol 8 /g dail5 for a chronic
1ipolar disorder with no /anic episodes for three 5ears4 Prior to that she
was hospitalized four ti/es per 5ear since age ;D for se.ere /anic
episodes4 -he has ne.er had a depression4 -he is now pregnant and
wishes to ha.e the 1a154
What would 5ou ad.ice her to do at this point6
<<
a5 *1tain an a1ortion( gi.en the fragilit5 of her condition and the significant
possi1ilit5 of another episodes as result of the pregnanc5 andBor
deli.er5
"5 -top all /edication
c5 -top the lithiu/ 1ut sta5 on the haloperidol
d5 -top the haloperidol and sta5 on the lithiu/
10.% In the treat/ent of /anic episodes( which of the following is '$-" useful
in the first three to four da5s of treat/ent6
a5 ntips5chotics
"5 'ithiu/
c5 Benzodizepine
d5 $)"
e5 quit( di/l5-lit roo/ to decrease sti/ulation
100% 28-5ear old has se.ere 1orderline personalit5 disorder /anifested 15
repeated self-har/ 1eha.ior( chronic sadness and e/ptiness( chronic
suicidal ideation and alcohol and cocaine a1use4
Which of the following is correct state/ent concerning this patients
treat/ent6
a5 Ps5chotherap5 would 1e directed at unco.ering the defense and
interpreting the /eaning of the acting out 1eha.ior
"5 "he use /edication should 1e a.oided
c5 "he focus of /anage/ent should 1e the pre.ention of suicide
d5 In ps5chotherap5 the therapist /a5 ha.e to accept suicide in the
patient as a possi1le outco/e
e5 long ter/ hospitalization is indicated( ai/ed at wor7ing through the
patientGs pri/iti.e defense /echanis/
102% Which of the following state/ents is correct a1out /ental retardation
a5 "he reported 0L pre.alence of /ental retardation is inaccurate
1ecause of culturall5 1iased IR tests
"5 Eorphological cere1ral patholog5 is e.ident in up to ?9L of those with
IRGs 1etween 89-?9
c5 Eost people with autis/ are also /entall5 retarded
d5 lthough recentl5 attaining a higher profile in the /edia( &ownGs
-5ndro/e is actuall5 a rare cause of geneticall5 deter/ined /ental
retardation
e5 +ertilit5 rates a/ongst the genetic t5pes of retardation are si/ilar to the
general population
;8?- in the elderl5( which of the following s5/pto/s is /ost suggesti.e of organic
Js non-
*rganic disorder6
a5 gitation
"5 Perser.ation
c5 &ecreased concentration
1>>
d5 path5
e5 Perplexit5
"%$ +*''*WIN, -"$E P$!"IN- "* R#$-"I*N- ;8D N& ;8@
00-5ear-old ga5 office wor7er co/plains of a four /onths histor5 of
Fanxiet5H4 %e clai/s the5 are discreet episodes that occur while on the
streetcar or at wor7( while sitting at his co/puter ter/inal4 "he feeling of
ner.ousness are acco/panied 15 sweating( tingling in the 1ac7 of his
head( a pounding heart( and shortness of 1reath( /ost s5/pto/s
su1sided after ;9 /inutes( with the exception of a /ild headache4
107% "he /ost li7el5 diagnosis is >
a5 gorapho1ia with panic
"5 *rganic anxiet5 s5ndro/e
c5 Panic disorder
d5 %5pochondriasis
e5 %istrionic personalit5 traits with secondar5 anxiet5
109% +or this patient( which of the following is the /ost appropriate
/anage/ent6
a5 Ps5chotherap5 exploring issues of sexualit5 and anxiet5
"5 Eedical wor7-up to/ rule out organic disease
c5 ,radual WW regarding his fear of ta7ing the streetcar
d5 I/ipra/ine
e5 Slorazepa/



1>1
ANSE!R TO MC"S
1% B 8;- & ;9;- $ ;8;- )
2% 82- B ;92- ) ;82- )
-% ) 80- ;90- ;80- )
.% B 8I- ;9I- ) ;8I- B
0% B 88- ) ;98- B ;88- )O&
2% & 8A- B ;9A- B ;8A- )
7% $ 8?- ) ;9?- ) ;8?- B
9% 8D- B ;9D- B ;8D- )
<% & 8@- ;9@- & ;8@- BO&
1>% ) A9- ;;9- $
11% A;- ;;;- $
12% ) A2- B ;;2- )
1-% $ A0- B ;;0- B
1.% B AI- ;;I-
10% A8- B ;;8-
12% & AA- $ ;;A- &
17% B A?- & ;;?- &
19% ) AD- ;;D-
1<% & A@- & ;;@- B
2>% ) ?9- & ;29- )
21% ) ?;- ) ;2;- B
22% ) ?2- ) ;22- &
2-% B ?0- $ ;20- B
2.% ) ?I- ;2I- B
20% ) ?8- & ;28- B
22% ) ?A- $ ;2A- )
1>2
27% & ??- & ;2?- B
29% ?D- & ;2D-
2<% ) ?@- B ;2@-
->% $ D9- B ;09-
-1% & D;- $ ;0;-
-2% & D2- B ;02-
--% ) D0- B ;00-
-.% B DI- B ;0I-
-0% D8- ;08-
-2% $ DA- B ;0A-
-7% & D?- ) ;0?-
-9% $ DD- ) ;0D-
-<% D@- B ;0@-
.>% $ @9- & ;I9-
.1% ) @;- & ;I;-
.2% $ @2- ) ;I2- )
.-% ) @0- ;I0- )
..% ) @I- B ;II- )
.0% ) @8- $ ;I8- BO&
.2% ) @A- ;IA- B
.7% & @?- B ;I?- &
.9% ) @D- ) ;ID- B
.<% @@- B ;I@- &
0>% ) ;99- & ;89- $
1>-

Você também pode gostar